>> Åwªï±z¡A³X«È¡G µn¤J½×¾Â «ö³o¸Ìµù¥U §Ñ°O±K½X ¦b½u·|­û ¤å³¹·j´M ½×¾Â­·®æ  ¨Ï¥Î»¡©ú   


>>> §ä¤H¿i¤ú©Î¥DÃD»Pª©­±µLÃöªº¹Ï¤å
¶ð¤º´Óª«¶é ¡÷ [ ´ú¸Õ°Ï ] [ªð¦^] ¡÷ ÂsÄý¡GSample cases¡@ ¼Ð°O½×¾Â©Ò¦³¤º®e¬°¤wŪ¨ú 

 ¥Ø«e½×¾ÂÁ`¦b½u 16 ¤H¡A¥»¥DÃD¦@¦³ 1 ¤HÂsÄý¡C¨ä¤¤µù¥U·|­û 0 ¤H¡A³X«È 1 ¤H¡C¡@ [Ãö³¬¸Ô²Ó¦W³æ]
µoªí¤@½g·s¥DÃD ¦^ÂФ峹 ¶}±Ò¤@­Ó·s§ë²¼ ¡»¦¹¤å³¹³Q¾\Ū 4764 ¦¸¡»¡@¡@ÂsÄý¤W¤@½g¥DÃD ¡@­«·s¾ã²z¥»¥DÃD  ¾ðª¬Åã¥Ü¤å³¹¡@ÂsÄý¤U¤@½g¥DÃD
 * ¤å³¹¥DÃD¡G Sample cases   ¥»¤å³¹¦³°ÝÃD¡A¶Ç°eµu°T®§³ø§iµ¹ª©¥D  ¥[¨ì§Úªº³Ì·R&Ãöª`¥»¤å³¹  Åã¥Ü¥i¦C¦Lªºª©¥»  §â¥»¤å³¹¥[¨ì§Úªº³Ì·R   
JuanFe 

 

¸ê®Æ: ¦¹·|­û¥Ø«e¤£¦b½u¤W Male ¨°Às ¥Õ¦Ï®y
«Â±æ: 0
¾y¤O: ¾y¤O: 78912
¸gÅç: ¸gÅç: 35968
¨Ó¦Û: ¦t©z¤¤¡@blank
µo¤å: 1118 ½g
ºëµØ: 0 ½g
¦b½u: 47¤Ñ19®É11¤À43¬í
µù¥U: 2013/06/17
Message ¬d¬Ý¡@·j´M¡@³q°T¿ý¡@½Æ»s¡@¤Þ¥Î¡@¦^ÂФ峹¦^ÂС@[¼Ó ¥D]
 [³o½g¤å³¹³Ì«á¥ÑJuanFe¦b 2018/09/13 08:08am ²Ä 7 ¦¸½s¿è]
1 Abdominal pain

2 Skin rash

3 Arm and leg weakness

4 Acute chest pain

5 Cough

6 fatigue and weight loss

7 diarrhea

8 sore throat

9 medication refill

10 constipation

11 importance

12 child fever

13 abdominal pain RUQ

14 prenatal visit

-------------------------------------------------------------------

15 shortness of breath

16 increase urination

17 jaundice

18 chest pain

19 abdominal pain RLQ

20 leg pain bilateral

21 vomiting

22 Acute chest pain

23 Frequent fall

24 Cough and chest pain

25 lower abdominal pain

26 Fatigue

27 hearing loss

28 Right knee pain

29 Blurred vision

-------------------------------------------------------------------

30 multiple bruises

31 burning during urination

32 difficulty swallowing

33 Refill medication for HIV

34 amenorrhea

35 right lower back and lower abdominal pain

36 insomnia

37 difficulty urination

38 panic attack

39 epigastric pain

40 hematemesis

41 dizziness

42 seizure

43 rectal bleeding







µoªí¤å³¹®É¶¡2018/06/05 11:03pm¡@IP: ¤w³]©w«O±K[¥»¤å¦@ 1087 ¦ì¤¸²Õ]¡@ 
JuanFe 

 

¸ê®Æ: ¦¹·|­û¥Ø«e¤£¦b½u¤W Male ¨°Às ¥Õ¦Ï®y
«Â±æ: 0
¾y¤O: ¾y¤O: 78912
¸gÅç: ¸gÅç: 35968
¨Ó¦Û: ¦t©z¤¤¡@blank
µo¤å: 1118 ½g
ºëµØ: 0 ½g
¦b½u: 47¤Ñ19®É11¤À43¬í
µù¥U: 2013/06/17
Message ¬d¬Ý¡@·j´M¡@³q°T¿ý¡@½Æ»s¡@¤Þ¥Î¡@¦^ÂФ峹¦^ÂС@[²Ä 2 ¼Ó]
 [³o½g¤å³¹³Ì«á¥ÑJuanFe¦b 2018/11/03 01:04pm ²Ä 1 ¦¸½s¿è]
1 Case 1 Scenario (abdominal pain)

Doorway information

The patient is a 30-year-old woman who comes to the clinic due to abdomen pain

Vital signs
. Temperature : 38.5¡¦C (101.3 F)
. Blood pressure : 120/75 mmHg
. Pulse : 98 /min
. Respiration : 22/ min

Basic differential diagnosis Gastrointestinal

. Appendicitis
. Acute cholecystitis
. Pancreatitis
. Inflammatory bowel disease . Irritable bowel syndrome
. Diverticulitis
. Bowel obstruction
. Acute gastroenteritis

Urinary

. Urinary tract infection / pyelonephritis
. Renal colic

Reproductive . Pelvic inflammatory disease
. Pelvic abscess
. Endometriosis
. Ovarian cyst / torsion
. Ectopic pregnancy
. Spontaneous abortion

Miscellaneous

. Shingles
. Aortic dissection

¡X¡X¡X¡X¡X

Case 1 sim. pt . instructions

If the doctor ask you about anything other then these , just say ¡§no¡¨ , or provide an answer that a normal patient might give.

You are a 30-year-old women who comes to the clinic with abdomen pain.

History of present illness

. The pain started 12 hours ago
. Started slowly , progressively increasing . ¡§Sharp¡¨ pain ; 6-7/10 in severity
. Felt all mover lower abdomen , but worst right below the umbilicus
. Began after eating a large meal
. Moving around makes it worse . No alleviating factors
. Fever since yesterday, with occasional chills today
. Intermittent nausea and vomiting . Passing urine more frequently and having burning on urination
. No bowel problems

Review of the systems

. Last menstrual period was 3 weeks ago . No discharge or abnormal bleeding form vagina
. Appetite and weight have not changed recently

Past medical /family /social history . One urinary tract infection in the past ; was serious and required hospitalization
. Current medications is an oral contraceptive pill only
. No allergies
. Immediate family members are all healthy
. Occupation : Receptionist ¡]±µ«Ý­û¡^
. Sexually active with multiple men
. Tobacco : no
. Alcohol : 2-3 drinks on weekend
. Recreational drugs: no

Ask this question : ¡§Doctor , is this appendicitis ?¡¨

¡X¡X¡X¡X¡X

Case 1 checklist

Following the encounter , check which of the following items were performed by examinee.

History of present illness / review of system

. Asked about the location of pain
. Asked about the intensity of pain
. Asked about the quality of pain
. Asked about the onset and duration of pain
. Asked about the course pf pain over time
. Asked about any radiation of pain
. Asked about any aggravating or relieving factors
. Asked about associated symptoms , especially :

1 Vomiting
2 Fever
3 Urinary problems
4 Bowel problems
5 Vaginal bleeding / discharge

. Asked about last menstrual period
. Asked about appetite and changes in weight

Past medical/ family / social history

. Asked about similar episodes in the past
. Asked about past medical issue , hospitalization , and surgeries
. Asked about current medications
. Asked about medication allergies
. Asked about family health
. Asked about tobacco, alcohol, and  drug use
. Asked about sexual and reproductive history
. Asked about occupation

Examination

. Examinee washed hands before examination
. Examined without gown , not through gown
. Auscultated abdomen
. Palpated abdomen(superficial and deep)
. Checked for rebound tenderness
. Checked for costovertibral angle tenderness
. Performed posts sign and obstructor sign

Counseling

. Explained physical findings and possible diagnosis
. Explained further workup
. Discussed safe sexual practices and sue of condom
. Asked to perform rectal and vaginal examination

Communication skills and professional conduct

. Knocked before entering the room
. Introduced self and greeted you warmly
. Used your name to address you
. Paid attention to what you said and maintained good eye contact
. Asked open-ended questions
. Asked non leading questions
. Asked one question at a time
. Listened to what you said without interrupting
. Used plain english rather than technical jargon
. Used appropriate transition sentences
. Used appropriate draping techniques
. Expressed empathy and made appropriated reassurances
. Asked whether you had any concerns/ questions

Differential diagnosis

. Pelvic inflammatory disease
. Pelvic abscess
. Urinary tract infection
. Appendicitis
. Ovarian torsion / rupture of cyst

Diagnostic study/ studies

. Rectal and vaginal examination
. CBC with differential count
. Urinalysis
. Pregnancy test
. Abdomen x-ray
. Abdomen ultrasound

¡X¡X¡X¡X¡X

Case 1 clinical summary

Clinical skill evaluation Case 1 patient note The following represents a typical note or this patient encounter . the details may vary depending on the information given by the simulated patient.

History : Described the history you just obtained from this patient, Include only information (pertinent positive and negatives ) relevant this patient¡¦s problem(s)

. 30-yo remake with 12 hours of abdominal pain
. Ate a heavy meal at a barbecue restaurant a day ago.
. Developed acute-onset periumbilical abdomen 12 hours later; described as a sharp , lasting a few minutes at a time , radiating diffusely in the abdomen , made worse with movement , and relieved with rest.
. Dysuria , increased urinary frequency , fever to 38.5¡¦C (101.3F) , chills , nausea ,and an episode of non bloody and non bilious vomiting.

ROS : No chest pain , shortness of breath , diarrhea , constipation
PMHx: Previous UTI with possible pyelonephritis
PSHx: None
Meds: None
Allergies : None FHx: noncontributory
SHx: HAs had 2 sexual partners with unprotected intercourse in the past month

Physical examination : Describe any positive and negative findings relevant to this patient¡¦s problem(s) . Be careful to include only those parts of the examination performed in this encounter

. Vital signsL Temperature , 38.5C (101.3F) ; blood pressure , 120/75 mmHg, pulse 98/min; and respiration¡¦s , 22/min
. Abdomen :Tenderness in the periumbilical , RLQ and LLQ regions . No abdomen dissension or guarding
. Normative bowel sounds
. Negative Murphy sign, Rovsing sign , posts sign , and obturator sign
. No CVA tenderness
. Lungs : Clear to auscultation
. Heart : Normal heart sounds with no murmurs

Data interpretation : Based on what you have learned form the history and physical examination, list up to 3 diagnoses that might explain this patient;s complaint(s) , List your diagnoses form most to least likely . For some cases , fewer than 3 diagnoses will be appropriate . Then , enter the positive or negative findings form the history and the physical examination (if present) that support each diagnosis. Lastly , list initial diagnostic studies (if any) you would order for each listed diagnosis (e,g, restricted physical examination maneuvers ,laboratory tests , imaging , ECG ,etc.).

Diagnosis #1 : Appendicitis

History findings

. Diffuse , abdomen pain
. Fever , chills
. Nausea ,vomiting

Physical Exam finding(s)

. Diffuse abdomen tenderness
. Fever

Diagnosis #2 : UTI

History findings

. Dysuria
. Increased urinary frequency
. Fever

Physical Exam findings

. Fever

Diagnosis #3 : Pelvic inflammatory disease

History findings

. Multiple sexual partners with unprotected intercourse
. Nausea, vomiting

Physical Exam findings

. Diffuse abdominal tenderness
. Fever

Diagnostic Studies

. Pregnancy test
. Urinalysis with culture
. CT scan of abdomen







µoªí¤å³¹®É¶¡2018/07/07 05:16am¡@IP: ¤w³]©w«O±K[¥»¤å¦@ 7892 ¦ì¤¸²Õ]¡@ 
JuanFe 

 

¸ê®Æ: ¦¹·|­û¥Ø«e¤£¦b½u¤W Male ¨°Às ¥Õ¦Ï®y
«Â±æ: 0
¾y¤O: ¾y¤O: 78912
¸gÅç: ¸gÅç: 35968
¨Ó¦Û: ¦t©z¤¤¡@blank
µo¤å: 1118 ½g
ºëµØ: 0 ½g
¦b½u: 47¤Ñ19®É11¤À43¬í
µù¥U: 2013/06/17
Message ¬d¬Ý¡@·j´M¡@³q°T¿ý¡@½Æ»s¡@¤Þ¥Î¡@¦^ÂФ峹¦^ÂС@[²Ä 3 ¼Ó]
 [³o½g¤å³¹³Ì«á¥ÑJuanFe¦b 2018/10/15 04:11pm ²Ä 3 ¦¸½s¿è]
2 case 2

Scenario (rash)

Doorway information about patient

The patient is a 27-year-old woman who comes to the office due to a rash.

Vital signs
. Temperature : 36.8¡¦C(98.3F)
. Blood pressure : 120/75 mmHg
. Pulse : 78/min
. Respirations: 16/min

Basic differential diagnosis

. Infections
- Bacterial (eg, cellulitis)
- Viral (eg,herpes zoster/shingles)
- Fungal (eg,tine corporis)
- Parasitic (eg, scabies)

. Psoriasis
. Acne vulgaris
. Rosacea
. Immune / autoimmune (eg, systemic lupus erythematous , erythema multiforme)
. Stasis dermatitis
. Bullous disorders (eg , bullous pemphigoid , dermatitis herpetiformis)

¡X¡X¡X¡X¡X

Case2 sim. pt. instructions

If the doctor asks you about anything other than these , just say ¡§ no¡¨ or provide an answer that a normal patient might give.

You are a 27-year-old woman who comes to the office with a rash

History of present illness

. the rash began 1 week ago
. Started after working in the garden
. Located on the face and neck
. Flat, with no bumps or blisters
. No associated itching , burning , pain
. Felt feverish but did not check temperature
. Rash getting larger but not spreading to other areas of the body
. Worse after going out on the sun, no alleviating factors
. No recent travel or sick contacts
. Also noticed joint pain and stiffness for about an hour in the morning , starting 4 days ago

Review of the system

. Last menstrual period was 2 weeks ago ; regular menses

Past medical /family/social history

. Intermittent joint pains in the past that resolve spontaneously ; never was evaluated by a doctor
. no medications except aspirin 7 days ago for a headache
. No surgeries
. No pregnancies
. Father is 55 and healthy ; mother is 54 and has ¡§rheumatism¡¨ ; sister is28 and has hypothyroidism
. Single . Works as a computer operator in a chemical manufacturing facility
. 1 sexual partner in the last month ; regular condom use
. Tobacco: No
. Alcohol: no
. Recreational drugs : No

Physical examination

. Skin: Multiple , well-circumscribed lesions on face and neck without vesicles ; no tenderness to touch

¡X¡X¡X¡X¡X

Case2 sim. pt checklist

Following the encounter , check which of the following items were performed by the examinee

History of present illness/review of system

. Asked about the location of rash
. Asked about whether the rash was initially flat or raised /blistered
. Asked whether the rash has changed over time or involved new areas
. Asked about any aggravating or relieving factors
. Asked about any causative factors
. Asked about associated symptoms , especially
- Itching or burning
- Pain
- Breathing problems or chest pain

. Asked about redness of eyes
. Asked about any joint pains
. Asked about fever
. Asked whether any close contacts have similar rash
. Asked about rennet travel
. Asked about any animal contact
. Asked about insect bites or outdoor activities in the recent past

Past medical / family / social history

. Asked about similar epodes of rash in the past
. Asked about past medical issue , hospitalizations ,a dn surgeries
. Asked about current medications
. Asked about medication allergies
. Asked about family health
. Asked bout tobacco, alcohol , and drug use
. Asked about sexual and reproductive history
. Asked about occupation

Examination

. Examinee washed heads before examination
. Examined without gown , not though gown
. Looked inside mouth for oral ulcers
. Examined hand joints
. Auscultate heart and lungs
. Examined face and neck for rash

Counseling . Explained  physical findings and possible diagnosis
. Explained further workup

Communication skills and professional conduct

. Knocked before entering the room
. Introduced self and greeted you warmly
. Used your name to address you . Paid attention to what you said and maintained good eye contact
. Asked open-ended questions . Asked non leading questions
. Asked one question at a time
. Listened to what you said without interrupting
. Used plain English rather than technical jargon

¡X¡X¡X¡X¡X

Clinical Skills evaluation

The following represents a typical note for this patient encounter . the details may vary depending on the information given by the simulated patient.

History : Describe the history you just obtained form this patient. Include only information (pertinent positive and negatives ) relevant to this patient¡¦s  problem(s)

. 27-yo female with 1 week of rash, . Gardened for 3 hours a week ago and developed a rash on her face and neck a few hours later . . Rash has increased in size but dose not involve other areas of the body.
. Morning joint pains and subjective fever.

ROS: no chest pain , shortness of breath , diarrhea , constipation , sick contacts , or recent travel PMHx : Episodes of joint pain and stiffness in the past , with spontaneous resolution
PSHx : None Meds : None Allergies : Noen FHx : Mother has possible rheumatoid arthritis.
SHx: HAs 1 sexual partner and uses condoms

Physical examination : Describe any positive and negative findings relevant to this patient¡¦s [problem(s) . Be careful yo include only those parts of the examination performed i this encounter.

. Vital signs : Temperature : 36.8¡¦C (98.3F) , blood pressure : 120/75mmHg , Pulse : 78/min, respirations : 22/min
. Face / neck : Multiple , well-circumscribes , erythematous macule without tenderness on palpation
. Skin : No clines , vesicles , or cysts in rash area . . Joints : Normal range of motion in all joints without tenderness , edema , or erythema . HEENT : no pallor , jaundice or eye lesion
. Lungs : CTA (C-lear T-o A-uscultate) bilaterally . Heart ; RRR without M/G/R

Data interpretation: Based in what you have learned form the history and physical examination , lists up to 3 diagnosis that might explain this patient¡¦s complaint(s) , List your diagnosis form most to least likely . For some cases , fewer than 3 diagnosis will be appropriate . then , enter the positive or negative diagnosis .Lastly , list initial diagnostic studies (if any) you would order for each listed diagnosis (eg , restricted physical exam maneuvers ,laboratory tests , imaging , ECG , eft.) .

Diagnosis #1 : SLE

History finding(s)

. Cutaneous photosensitive rash
. Fever
. Joint pain

Physical Examination finding(s)

. Discoid lupus rash

Diagnosis #2 : Rheumatoid arthritis

History findings

. Joint pain and morning stiffness
. Fever
. Family history

Physical Exam findings

. None

Diagnosis #3 : Photodermatitis

History finding(s)

. History of sun exposure followed by rash Physical Exam finding(s)

. Photosensitive rash on face and neck
. No progression of rash to other areas

Diagnosis Studies . ANA and anti-ds DNA
. Rheumatoid factor and ESR
. Skin biopsy








µoªí¤å³¹®É¶¡2018/07/07 05:19am¡@IP: ¤w³]©w«O±K[¥»¤å¦@ 7173 ¦ì¤¸²Õ]¡@ 
JuanFe 

 

¸ê®Æ: ¦¹·|­û¥Ø«e¤£¦b½u¤W Male ¨°Às ¥Õ¦Ï®y
«Â±æ: 0
¾y¤O: ¾y¤O: 78912
¸gÅç: ¸gÅç: 35968
¨Ó¦Û: ¦t©z¤¤¡@blank
µo¤å: 1118 ½g
ºëµØ: 0 ½g
¦b½u: 47¤Ñ19®É11¤À43¬í
µù¥U: 2013/06/17
Message ¬d¬Ý¡@·j´M¡@³q°T¿ý¡@½Æ»s¡@¤Þ¥Î¡@¦^ÂФ峹¦^ÂС@[²Ä 4 ¼Ó]
 3 Case 3

Scenario (arm and leg weakness)

Doorway information about patient

The patient is a 65-year-old woman who comes to the emergency department due to are and leg weakness.

Vital signs
. Temperature
. Blood pressure : 160/90 mmHg
. Pulse : 78/min
. Respirations : 22 /min

Basic differential diagnosis Neurologic . Stroke . Transient ischemic attack (TIA)
. Subarachnoid hemorrhage
, Subdural hematoma
. Intracranial mass
. Guillain -Barre syndrome . Spinal cord lesion . Complex migraine

Metabolic . Hypoglycemia . Hypothyroidism
. Adrenal insufficiency
. Electrolyte disorders

Musculoskeletal

. Myopathy Miscellaneous

. Conversion disorder
. Heart-Reflated illness

¡X¡X¡X¡X¡X

Case 3 sim. pt. instructions

If the doctor asks you about anything other than  these , just say ¡§ no ¡§ , or provide an answer that a normal patient might give .

You are a 65-year-old woman who comes to the emergency department with 1 hour of right arm and leg weakness.

History of illness . The symptoms started an hour ago with weakness in the right  arm and leg
. Gradually increasing numbness
. Moderate (5/10 in severity) headache that felt¡¨all over ¡§ the head
. Nausea but no vomiting
. No slurred speech or difficulty swallowing
. No blurred or double vision.
. No recent fall or loss consciousness . no symptoms like this in the past.

Review of systems

. No fever
. No chest pain or palpitations
. No diarrhea . No urinary symptoms
. No seizures

Past medical/family/social history

. High blood pressure for 25 years
. High cholesterol
. Heart attack 6 years ago; heart bypass surgery at that time
. Medications : Simvastatin 20 mg daily, aspirin 81mg daily, atenolol 50 mg daily
. Medication allergies : None
. Both parents had hypertension and died in their 60s of heart attacks
. Widow (husband died 8 years ago); lives alone
. Bought to the hospital by neighbor (¡§Steve¡¨) who si closet contact and is ¡§Like a son to me¡¨
. Tobacco: 2 pack of cigarettes a day for 35 years and quit 6 years ago
. Alcohol : 1-2 drinks , once a month
. Recreational drugs : None

Physical examination

Neurological:
. Weaker on the right side of the body
. Unable to lift right leg or arm without assistance
. Unable to stand . Cranial nerves are normal . Reflexes are slightly exaggerated on the right
. Babinski: Upping on right and downgoing on the left
. Sensation is normal on both sides of body

The rest of the examination is normal

Ask this question : If the examinee dose not discuss the possibility of a stroke , ask , ¡§Doctor , is it a stroke?¡¨

¡X¡X¡X¡X¡X

Cases3 sim. pt. checklist

Following the encounter , check which of the following items were performed by the examinee.

History of present illness /review of systems

. Asked about the onset of weakness
. Asked where the weakness is felt
. Asked if the weakness is changing over time
. Asked how you felt prior to onset of the weakness
. Asked about associated symptoms

- Sensory changes or numbness
- Loss of consciousness
- Seizures/jerky movements
- Fever
- Nausea/vomiting
- Chest pain, palpations
- Problems with speech or swallowing
- Visual changes (eg , blurred vision, double vision)
- Incontinence / Bowel or bladder dysfunction

. Asked about a history of frequent fall/spells
. Asked about any history if recent head trauma

Past / family/ social history

. Asked about similar symptoms in the past
. Asked about past/other medical issues (especially hypertension, diabetes mellitus , hypercholesterolemia, myocardial infarction, strokes ,migraine headaches)
. Asked about previous hospitalizations and surgeries
. Asked about current medications
. Asked about medication allergies
. Asked about any family history of stoke , heart attacks , or aneurysms
. Asked about alcohol intake
. Asked about living situation

Examination

. Examinee washed hands before examination
. Examined without gown , not though gown
. Checked cranial nerves II-XII
. Tested muscle power bilaterally
. Checked deep-tendon reflexes in bother the upper / lower extremities
. Checked for sensory modalities proximally and distally
. Checked coordination and gait
. Listened for carotid bruits
. Checked for neck stiffness
. Auscultated heart

Counseling

. Explained the physical findings and possible diagnosis
. Explained further workup

Communication skills and professional conduct

. Knocked before entering the room
. Introduced self and greeted you warmly
. Used your name to address you
. Paid attention to what you said and maintained good eye contact
. Asked open-ended questions
. Asked non-leading questions
. Asked one question at a time
. Listened to what you said without interrupting
. Used plain English rather than technical jargon
. Used appropriated transition sentences
. Used appropriate draping techniques
. Expressed empathy and made appropriate reassurances
. Asked whether you had any concerns./questions

Differential diagnosis

. Stroke
. Transient ischemic attack . Subarachnoid hemorrhage

Diagnosis study . CBC with differential . Basic metabolic panel (or glucose and electrolytes)
. CT scan of the head without contrast
. Doppler ultrasound of the carotid arteries . ECG
. Transesophageal echocardiogram

¡X¡X¡X¡X¡X

Clinical Skill Evaluation
Case 3 Patient Note

The following represents a typical note for this patient encounter .  the details may vary depending on the information given by the simulated patient.

History: Describe the history you just obtained from this patient. Include only information (patent positives and negatives) relevant to this patient¡¦s problem(s)

. 65-yo female with an hour of acute-onset , right -sided weakness and headache.
. Gradually progressing symptoms over the pats hour.
. Nausea without vomiting.
. No history of fall or syncope.

ROS: no fever , chest pain , shortness of breath , vision changes , dysarthria , seizures
Max : HTN , hypercholesterolemia , CAD
PSHx : CABG 6 years ago
Meds : Aspirin , simvastatin , atenolol
Allergies : None FHx : Mother and father had HTN and died of MI , Brother has HTN and hypercholesterolemia
SHx : Smoked 2 PPD for 35 years but quit 6 years ago

Physical examination : Describe any positive and negative findings relevant to this patient¡¦s problem(s). Be careful to include only those parts of the examination performed in this encounter.

. Vital signs : Temperature : 36.8¡¦C (98.2 F) , Blood pressure : 160/90 mmHg , pulse : 78 /min , respirations : 16/min.
. Lungs :  CTA bilaterally
. Heart : RRR without M/G/R
. Neurological ; A&Ox3 , CN 2-12 intact , motor 5/5on LUE and LLE but 3/5 in RUE and RLE , sensory grossly intact , DTR 2+ on left but 3+ in RUE and RLE , upping toes on right and downing on left , gait unable to be assessed.

Data interpretation: Based on what you have learned from the history and physical examination ., .lsit up to 3 diagnosis that might explain this patients complaint(s) , list your diagnosis form most to least likely . for some cases , fewer than 3 diagnosis will  be appropriate . then , enter the positive or negative findings form the history and physical examination (if present) that support each diagnosis . lastly ,list initial diagnostic studies (if any ) you would order for each listed diagnosis (eg , restricted physical examination maneuvers , laboratory tests , imaging , ECG , etc.).

Diagnosis #1 ; Evolving stroke

History finding(S)

. Acute -onset weakness
. Gradually progressing symptoms Physical Exam finding(s)

. Right hemiparesis
. Eight-side hyperflexia
. Right Babinski rifles present

Diagnosis #2: TIA or reversible ischemic neurological deficit

History finding(s)

. Acute-onset weakness

Physical Exam finding(S)

. Right hemiparesis
. Right-sided  hyperreflexia
. Right Babinski reflex present

Diagnosis #3 : Subarachnoid hemorrhage

History finding(s)

. Headache
. Nausea
. Acute-onset weakness

Physical Exam finding(s)

. Right hemiparesis
. Right-sided hyperreflexia
. Right Babinski reflex present

Diagnosis Studies

. CT scan of head without contrast
. Transesphageal echocardiogram
. Carotid Doppler
. CBC with differential







µoªí¤å³¹®É¶¡2018/07/07 05:20am¡@IP: ¤w³]©w«O±K[¥»¤å¦@ 8595 ¦ì¤¸²Õ]¡@ 
JuanFe 

 

¸ê®Æ: ¦¹·|­û¥Ø«e¤£¦b½u¤W Male ¨°Às ¥Õ¦Ï®y
«Â±æ: 0
¾y¤O: ¾y¤O: 78912
¸gÅç: ¸gÅç: 35968
¨Ó¦Û: ¦t©z¤¤¡@blank
µo¤å: 1118 ½g
ºëµØ: 0 ½g
¦b½u: 47¤Ñ19®É11¤À43¬í
µù¥U: 2013/06/17
Message ¬d¬Ý¡@·j´M¡@³q°T¿ý¡@½Æ»s¡@¤Þ¥Î¡@¦^ÂФ峹¦^ÂС@[²Ä 5 ¼Ó]
 [³o½g¤å³¹³Ì«á¥ÑJuanFe¦b 2018/08/15 03:39pm ²Ä 1 ¦¸½s¿è]
4 Case 4 scenario (acute chest pain)

Doorway Information About patient

The patient is a 29-year-old woman with known sickle cel anemia who comes to the emergency department due to chest pain

Vital signs

. Temperature : 38.5¡¦C (101.3F)
. Blood pressure: 120/75 mmHg
. Pulse : 110/min
. Respirations : 22/min

Clinical images Electrocardiogram T-S slightly upward ?

Basic differential diagnosis Hematologic

. chest syndrome due to sickle cell anemia

Cardiovascular

. Pericarditis
. Acute coronary syndrome

Pulmonary

. Pneumonia
. Pulmonary thromboembolism

Musculoskeletal

. Costochondritis
. Salmonella osteomyelitis

Other

. Panic attack

¡X¡X¡X¡X¡X

Case 4 sim pt. instructions if the doctor asks you about anything other than these , just say ¡§ no, ¡¥ or provide an answer that a normal patient might give.

You are 29-year -old women who comes to the emergency department with chest pain.

History of present illness

. Pain started 12 hours ago
. Pain located in the middle of the chest and dose not move.
. Pain started slowly bit is increasing and is now 7-8/10 in severity.
. Pain worse with any movement or deep breathing.
. Took acetaminophen , which reduced the pain slightly.
. Fever as high as 38.3¡¦C (101F) . caught with green sputum , and imild shortness of breath for the last 3 days.
. No blood in the sputum or recent chest trauma.

Review of the systems

. No used , vomiting , diarrhea , or abdominal pain.
. No leg pain or swelling
. No back pain
. No urinary symptoms
. Last menstrual period was 2 weeks ago

Past medical / family / social history

. Sickle cell anemia: Diagnosed in childhood ; had mild pain episodes at that time. 1 transfusion(age 18). Admitted once for abdominal pain 5 years ago that resolved with supportive care in hospital.
. No surgires
. No pregnancies
. Medications : Birth control pill , acetaminophen as needed
. Allergies ; No drug allergies
. Mother also has sickle cell anemia; brother and father are healthy
. Occupation ; Teacher
. Single , sexual active with 1 boyfriend and use condoms regularly
. Tobacco : No . Alcohol : No
. Recreational drugs : No

Physical examination

Head and neck: . No redness or exudates in the mouth /pharynx
. No enlarged lymph nodes

Chest/lungs:

. No tenderness to palpation of the chest wall
. Clear to auscultation bilaterally

Heart; . Regular rhythm with borderline tachycardia
. No murmurs

Abdomen:

. Nontender , non distended
. Normative bowel sound throughout
. Tympanic to percussion
. No hepatosplenomegaly

Extremities:

. No cyanosis . No tenderness in the legs

You should also try to breath a little faster than normal to rate near 20/min . the rest of the examination is normal.

¡X¡X¡X¡X¡X

Case 4 sim. pt. checklist

Following the encounter , check which of the following items were performed by the examinee.

History of present illness/ review of the system

. Asked about the location of pain
. Asked about the intensity of pain
. Asked about the quality of pain
. Asked about the origin and duration off pain
. Asked about the progression of pain
. Asked about any radiation of pain
. Asked about any radiation of pain
. Asked about the aggravating/ relieving factors
. Asked about associated symptoms , especially:
- Nausea
- Fever
- cough
- Shortness of breath
. Asked about precipitating gators of sickle cell crisis (eg, diarrhea a, dehydration , stress)
. Asked boy history of chest trauma
. Asked about nay leg pain / swelling / redness
. Asked about blood in the urine
. Asked bout prior blood transfusion

Past medical / family / social history

. Asked about any similar problems in the past
. Asked about other past medical issues
. Asked about previous hospitalizations and surgeries
. Asked about menstrual / reproductive history
. Asked about current medications (prescription and over the counter)
. Asked about any medication allergies
. Asked about family history of sickle cell anemia , heart problems , and blood clots
. asked about tobacco , alcohol , and drug use
. Asked about occupation

Examination

. Examinee chased hands before examination
. Examined without gown , not through gown
. Examined the oral cavity
. Examined for enlarged lymph nodes
. Performed inspection and palpation of the chest
. Performed palpation of area
. Auscultated the lungs and heart
. Palpated abdomen (superficial and deep)
. Examined hands and fingers
. Examined legs for tenderness

Counseling . Explained the physical findings and possible diagnosis
. Explained the complications of sickly cell disease (infections , hypoxia can precipitate pain)
. Exclaimed further workup
. Discussed the importance of avoiding hypoxemia and maintaining hydration

Communication skills and professional conduct

. Knocked before entering the room
. Introduced self and greeted you warmly
. Used your name to address you
. Paid attention to het you said and maintained good eye contact
. Asked few open-ended questions . Asked non-leading questions . Asked one question at a time
. Listened to what you said without interrupting
. Used plain English rather than technical jargon
. Used appropriate transition sentences
. Used appropriate draping techniques
. Expressed empathy and made appropriate reassurances

Different diagnosis

. Chest syndrome due to sickle cell anemia
. Pneumonia
. Costochondritis
. Pericarditis
. Pulmonary thromboembolism
. Salmonella osteomyelitis

Diagnostic study / studies

. CBC with differential . Urinalysis
. Sputum Gram and culture
. Blood cultures
. Chest x-ray
. ECG (provided with doorway information)

¡X¡X¡X¡X¡X

Case 4 clinical summary

Clinical Skill Evaluation
Case 4 Clinical Note

The following represents a typical note for this patient encounter. The details may vary depending on the information given by the simulated patient.

History ; Describe the history you just obtained form this patient o. Include only information (pertinent positive and negatives) relevant to tis patient¡¦s problem(s).

. 29-yo female with history of sickle cell anemia and 12 hours of acute -onset chest pain.
. Grade; increase in pain to 7-8 on a scale of 10 over the past 12 hours.
. Pain is worse with movement and respiration and improved with acetaminophen.
. 3 days of fever to 38.3¡¦C (101F) , cough productive of green sputum , and mild shortness of breath.

ROS: No nausea , vomiting, trauma to chest , dysuria , diarrhea , leg swelling , or leg pain.
PMHx : Admitted once 5 years ago for abdominal pain that resolved
PSHx : None
Meds : Birth control pills , Tylenol PRN
Allergies : Noen
FHx : Mother has sickle cell anemia
SHx : Denies use of tobacco, alcohol , and elicit drugs , Had a blood transfusion as a child.
ECG shows sinus tachycardia with no diagnostic abnormalities

Physical examination:
Describe any positive and negative findings relevant to this patient¡¦s problem(s). Be careful to include only those parts of the examination performed in this encounter.

. Vital signs : Temperature , 38.5¡¦C (101¡¦3F); blood pressure , 120/75 mmHg; pulse , 110/min; and respirations , 2/min
. HEENT : Oropharynx without erythema or exudates
. Neck : No cervical lymphadenopathy , trachea midline
. Lungs : CTA bilaterally , no chest tenderness to palpation
. Heart : RRR without M/R/G
. Abdomen : Contender , non distended , normative bowel sounds thought ; tympanic to percussion , no hepatosplenomegaly
. Extremities : No cyanosis, clubbing , or edema ; no swelling or tenderness in the legs

Data interpretation; Based on what you have learned form the history and physical examination, List up to 3 diagnosis that might explain this patient¡¦s complaints(s). List your diagnosis form poset to least likely . For some cases , fewer than 3 diagnosis will be appropriated  then , enter the positive ro negative findings form the history and the physical examination (if present) that support each diagnosis . Lastly ,list initial diagnostic studies (if nay) you would order for each listed diagnosis(e.g., restricted physic examination maneuvers, laboratory testes , imaging , ECG, etc)

Diagnosis #1 : Acute chest syndrome

History finding(s)
. Acute -onset chest pian
. Shortness of breath
. History of sickle cell anemia

Physical Exam finding(s)
. No chest pain or palpation
. Fever

Diagnosis#2 : Pneumonia History finding(s)
. Fever . Cough productive of green sputum
. Shortness of breath

Physical Exam finding(s)
. Fever
. Tachypnea

Diagnosis#3 ; Pulmonary embolism

History finding(s)
. Pleuritic chest pain
. Fever . Shortness of breath
. Birth control pills

Physical Exam finding(s)
. Fever . No chest tenderness ot palpation

Diagnostic Studies
. chest x-ray
. ABG
. Blood cultures
. CBC with differential








µoªí¤å³¹®É¶¡2018/07/31 03:21am¡@IP: ¤w³]©w«O±K[¥»¤å¦@ 9310 ¦ì¤¸²Õ]¡@ 
JuanFe 

 

¸ê®Æ: ¦¹·|­û¥Ø«e¤£¦b½u¤W Male ¨°Às ¥Õ¦Ï®y
«Â±æ: 0
¾y¤O: ¾y¤O: 78912
¸gÅç: ¸gÅç: 35968
¨Ó¦Û: ¦t©z¤¤¡@blank
µo¤å: 1118 ½g
ºëµØ: 0 ½g
¦b½u: 47¤Ñ19®É11¤À43¬í
µù¥U: 2013/06/17
Message ¬d¬Ý¡@·j´M¡@³q°T¿ý¡@½Æ»s¡@¤Þ¥Î¡@¦^ÂФ峹¦^ÂС@[²Ä 6 ¼Ó]
 [³o½g¤å³¹³Ì«á¥ÑJuanFe¦b 2018/08/15 04:00pm ²Ä 2 ¦¸½s¿è]
5 Case 5 scenario (cough)

Doorway information about patient

The patient is a 35-year -old man who comes to the office due to cough

Vital signs
. Temperature : 38.5¡¦C(101.3F)
. Blood pressure : 120/75 mmHg
. Pulse : 98/min
. Respirations :  20/min

Basic differential diagnosis
. Common cold
. Acute sinusitis
. Allergic rhinitis
. Acute bronchitis
. Pneumonia
. Pertussis
. Pulmonary embolism
. Drugs (ACE inhibitors)
. Asthma
. Congestive heart failure
. Chronic obstructive pulmonary disease (COPD) exacerbation

¡X¡X¡X¡X¡X

Case 5 sim. pt. instructions

f the patient asked you about anything other than these , just say ¡§ no ¡§ , for provide an answer that a normal patients might give.

You are a 35-tear-old man who  comes to the office with a cough.

History of present illness:

. the cough started 5 days ago
. Yellow sputum (¡§ teaspoon size¡¨ at a time)
. Cough is worse at n eight and is keeping you awake.
. Other symptoms include sore throat , sinus congestion , running nose, mild frontal headache
. No blood in the sputum , chills m night sweats , chest pain , shortness of breath, or wheezing
. Symptoms slightly better with over-the -counter cough medicine (guaifenesin/dextromethorphan)
. Temperature at home was 37.9¡¦C(100.2F)
. 8 year-old son was recently sick with similar symptoms

Past medical/family/social history:

. Asthma l had it since childhood with mild intermittent symptoms
. Seasonal allergies; mainly in the fall and spring
. No surgieres
. Medications ; Albuterol inhaler as needed (2-3 times a month)
. Allergies : No medication allergies , but allergic to cats
. Father is 60 and has asthma ; mother is 59 and is healthy ; no siblings
. Married , live with spouse and 2 children
. Occupation : Paramedic
. Tobacco : 1 pack per day for 10 years
. Alcohol : 1-2 drinks on social occasions
. Recreational drugs : None

Physical examination

Head and neck :
. No redness or exudates in the mouth
. No sinus tenderness to percussion
. No enlarged lymph nodes

Chest/lungs:
. Clear to auscultation bilaterally

heart: . Regular rate and rhythm; No murmurs

You should also try to breath a little faster to a rate near 20 /min . the rest of the examination is normal

¡X¡X¡X¡X¡X

Case 5 sim. pt. checklist

Following the encounter , check which of the following itms were performed buy the examinee.

History of present illness/ review of systems

. Asked about the onset of cough
. Asked about the duration of cough
. Asked about whether cough is dry or productive
. Asked for descriptions of  sputum(color, quantity)
. Asked about any problem with breathing
. Asked about wheezing
. Asked about associated symptoms , especially:
- Chest pain
- Fever and chills
- Sinus congestion/pain
- Running nose
- Sore throat
. Asked about contacts with ill persons(workplace and home)

Past medical / family / social history

. Asked about similar episodes in the past
. Asked about past medical issues(asthma , chronic allergies)
. Asked about previous hospitalization and surgeries
. Asked about medications (prescription and over the counter)
. Asked about allergies
. Asked about family health
. Asked about occupation
. Asked about tobacco , alcohol, and drug use

Examination

. Examinee washed hands
. Examined without gown , not though gown
. Examined nose and throat
. Checked neck for lymph nodes
. Palpated sinus
. Listened to lungs
- Asked you to say 99 repeatedly(palpated for tactile vocal fremitus).
. Tapped on lungs (percussion)
. Auscultated heart

Counseling

. Explained physical findings and possible diagnosis.
. Emplane the further workup
. Discussed quitting smoking
. Discussed simple measures for comfort (eg, over the counter medications, fluids , humidifier air)

Communication skills and professional conduct

. Knocked before entering the room
. Introduced self and greeted you warmly
. Used your name to address you
. Paid attention to what you said and maintained good eye contact
. Asked few open-ended questions
. Asked non-leading questions
. Asked one question at a time
. Listened to what you said without interrupting
. Used plain english reader than technical jargon
. Used appropriate transition sentences
. Used appropriate draping techniques
. Summarized the history and explained physical findings
. Expressed empathy and gave appropriate reassurances
. Asked whether you had any concerns / questions

Differential diagnosis

. Common cold
. Acute sinusitis
. Acute bronchitis
. Pneumonia

Diagnostic study/studies

. CBC with differential count
. Sputum Gram stain and culture
. Chest x-ray (posterior anterior and lateral view)

¡X¡X¡X¡X¡X

Case 5 clinical summary

Clinical Skills Evaluation
Case 5 Patient Note

The following represents a typical note for this patient encounter . the details may vary depending on the information given by the simulated patient.

History : Describe the history you just obtained form this patient. Include only information (pertinent positives and negatives) relevant to this patient¡¦s problem(s).

. 35-yo male with 5 days of worsening cough
. Sore throat, fever , rhinorrhea, and frontal headache.
. Cough productive of approximately 1 tsp of yellow sputum without hemoptysis
. recent sick contact : 8-yo son.

ROS : No chills , night sweats , chest pain , shortness of breath , wheezing , or abdominal pain
PMHxL Mild asthma, seasonal allergic  rhinitis.
PSHx: None
Meds ; Albuterol MDI PRN
Allergies: Cats
FHx : Father has asthma
SHx : has smoked 1 PPD for past 10 years and drinks occasionally

Physical examination :
Describe any positive and negative findings relevant to this patient¡¦s problem(s). be careful in include only those parts of the examination performed in this encounter.

. Vital designs: Temperature , 38.3¡¦C(101F) ; blood pressure , 120/75 mmHg ; pulse, 98/min ; restorations , 20/min
. HEENT : Oropharynx without erythema or exudates. no sinus tenderness to percussion
. Neck : No cervical lymphadenopathy , trachea midline
. Lungs; CTA bilaterally
. Heart : RRR without M/R/G

Data interpretation: Based on what you have learned form the history and physical examination, list up to 3 diagnosis that might explain this patient¡¦s complaint(s). List your diagnosis from most to least likely . for some cases , fewer than 3 diagnoses will be appropriate . Then , enter the positive or negative findings form he history and the physical examination (if present) that support each diagnosis. Lastly , list initial diagnostic studies (if nay) you would order for each listed diagnosis (e.g.,restricted physical exam maneuvers , laboratory tests, imaging , ECG , etc.).

Diagnosis #1 : Acute sinusitis

History finding(s)
. History of allergic rhinitis
. Frontal headache
. Fever

Physical Exam finding(s)
. Fever

Diagnosis # 2 : Pneumonia

History finding(s)
. Fever
. Cough productive of yellow sputum
. Smoking history

Physical Exam finding(s)
. Fever
. Tachypnea

Diagnosis # 3 : Acute bronchitis

History finding(s)
. Cough
. Fever
. Smoking history

Physical Exam finding(s)
. Fever

Diagnostic Studies
. Chest x-ray
. Sputum Gram stain and cultures
. CBC with differential







µoªí¤å³¹®É¶¡2018/07/31 03:22am¡@IP: ¤w³]©w«O±K[¥»¤å¦@ 7660 ¦ì¤¸²Õ]¡@ 
JuanFe 

 

¸ê®Æ: ¦¹·|­û¥Ø«e¤£¦b½u¤W Male ¨°Às ¥Õ¦Ï®y
«Â±æ: 0
¾y¤O: ¾y¤O: 78912
¸gÅç: ¸gÅç: 35968
¨Ó¦Û: ¦t©z¤¤¡@blank
µo¤å: 1118 ½g
ºëµØ: 0 ½g
¦b½u: 47¤Ñ19®É11¤À43¬í
µù¥U: 2013/06/17
Message ¬d¬Ý¡@·j´M¡@³q°T¿ý¡@½Æ»s¡@¤Þ¥Î¡@¦^ÂФ峹¦^ÂС@[²Ä 7 ¼Ó]
 6 Case 6 scenario (fatigue and weight loss)

Door way information about patient

The patient is a 50-year-old man who comes to the office due to fatigue and weight loss.

Vital signs
. Temperature : 36.7¡¦C (98.1F)
. Blood pressure : 120/76 mmHg
. Pulse : 78 /min
. Respirations : 18 /min

Basic differential diagnosis

Infection
. HIV . Tuberculosis

Metabolic disorders
. Diabetes
. Thyroid disorder
. Adrenal insufficiency

Malignancy
. Solid tumor
. Hematologic malignancy

Gastrointestinal . Hepatitis
. Malabsorption

Other
. Depression
. Eating disorder
. Medication side effect

¡X¡X¡X¡X¡X

Case 6 sim. pt. instruction

If the doctor asks you about anything otters than these , just say ¡§no,¡¨ or provide an answer that a normal patient might give.

You are a 50-year-old man who comes to the office with fatigue

History of present illness

. The symptoms stated 5 months ago
. Symptoms were initially mild but have been worse over the last 3 months
. Generalized weakness but no focal weakness
. Symptoms are associated with intermittent ¡§ gas pain ¡§ around the umbilicus
. Feel full after eating only small meals

(Include the following information only of asked what may have caused / trigged your symptoms.)

. Symptoms got worse after your spouse died 3 months ago
. Decreased appetite with a 13.6-kg(30-lb) weight loss
. Decreased interest in activities
. Difficulty falling asleep at night ; also waking up frequently at night and unable to get back to sleep
. No thoughts about suicide , but have feelings of guilt
. Difficulty concentrating on tasks
. Thinking that your family dose not understand what you are going though and feeling isolated form many of your friends

Review of systems . No fever , chills
. No nausea, vomiting , diarrhea , or constipation
. No chest pain, or shortness of breath
. No jaundice
. No numbness, tingling , or tremor

Past medical / family / social history

. No prior medical problems
. No surgeries
. No medications
. No drug allergies
. Mother died at age 60 of pancreatic cancer ; after died at age 55 of heart attack ; no siblings
. Widower, living aloe
. 2 children (ages 28, 25)
. Occupation : restaurant manager
. Tobacco: No
. Alcohol : 2-3 drinks on social occasions
. Recreational drugs: No

Physical examination

Head and neck:
. No readiness or exudates in the mouth
. No enlarged lymph nodes
. No thyromegaly

Chest / Lungs : . No tenderness to palpation of the chest wall
. Clear to auscultation bilaterally

Heart:
. Regular rate and rhythm
. No murmurs

Abdomen:
. Non tender, non distended
. Normative bowel sounds throughout
. Tympanic to percussion
. No hepatoslenomegaly
. No jaundice

Extremities:
. No cyanosis , clubbing , edema

Neurological:
. Normal motor strength and deep-tendon reflexes

¡X¡X¡X¡X¡X

Case 6 sim. pt. check list

Following the encounter , check which  of the following items were performed by the examinee

History of present illness/review of the system

. Asked  about the onset and progression of weakness. fatigue (open-ended question)
. Asked about associated symptoms , especially:
- Fever , chills , night sweats
- Enlarged lymph nodes
- Temperature intolerance (hot or cold)
- Chest pain, cough , and shortness of breath
- Nausea and vomiting
- Change in appetite and weight
- Difficulty swallowing
- Abdominal pain
- Jaundice
- Blood in stools or black stools
- Insomnia/sleep
. Enquired about any precipitating factors
. Asked about mood/emotional state
. Asked about interest in life
. Asked about any guilt feelings
. Asked about any ideas , plans , attempts for suicide

Past medical / family / social history

. Asked about similar episodes in the past
. Asked about past medical issue
. Asked about previous hospitalization and  surgeries
. Asked about medications
. Asked about medication allergies
. Asked about family health
. Asked about occupation
. Asked about tobacco, alcohol, and drug use

Examination

. Examinee washed hands
. Examined without gown , not though gown
. Examined eyes
. Examined oral cavity
. Examined neck for thyromegaly and lymphadenopathy
. Auscultated test (heart and lungs)
. Palpated abdomen , both superficially and deeply
. Checked leg for edema
. Check muscle power
. Looked for ankle jerk / reflex

Counseling

. Explained the physical findings and possible diagnosis
. Explained further workup
. Inquired regarding need for any additional emotional support

Communication skills and professional conduct

. Knocked before entering the room
. Introduced self and greeted you warmly
. Used your name to address you
. Paid attention to what you said and maintained good eye contact
. Asked open-ended questions
. Asked non leading questions
. Listened to what you said without interrupting
. Used plain English rather than technical jargon
. Used appropriate draping techniques
. Summarized the history and explained physical findings
. Expressed empathy and gave appropriate reassurances
. Asked whether you had any concerns/ questions

Differential diagnosis

. Occult malignancy
. Hyper/hypothyroidism
. Depression

Diagnostic study/ studies

. Rectal examination and stool examination for occult blood
. CBC with differential
. Glucose and electrolytes
. TSH
. Liver function tests

¡X¡X¡X¡X¡X

Case 6 clinical summary

Clinical Skill Evaluation
Case 6 Patient Note

The following represents a typical note for this patient encounter . the details may vary depending on the information given by simulated patient

History : Describe the history you just obtained form this patient. Include only information (pertinent positive and negatives) relevant to this patient¡¦s problem(s).

. 50-yo male with 5 months of increased fatigue.
. Spouse died 3 months ago and symptoms have worsened since then.
. 13.6-kg (30-lb) weight loss, decreased appetite , periumbilical abdominal pain , early satiety.
. Loss of interest in activities and terminal insomnia but not suicidal

ROS: No dysphagia , fever , chills , night sweats , chest pain , shortness of breath, or cough
PMHx: None
PSHx: None
Meds: None
Allergies: None
FHx: Mother died at age 60 form pancreatic cancer , after died at age 55 form heart attack
SHx: Denies tobacco use

Physical examination: Describe any positive and negative findings relevant to this patient¡¦s problem(s). be careful to include only those parts of the examination performed in this encounter.

.Vital signs : Temperature, 36.7¡¦C (98F) ; blood pressure , 120/76 mmHg ; pulse , 78/min : and respirations , 18/min
. Head / neck : No redness or exudates in the mouth , no enlarged lymph nodes , no jaundice , no thyromegaly
. Chest /lungs: No tenderness to palpation of the chest wall , clear to auscultation bilaterally
. Heart : RRR
. Abdomen : Non tender , non distended , normative bowel sounds throughout ; tympanic to percussion ; no hepatosplenomegaly
. Extremities : No cyanosis , clubbing, or edema
. Neurological : Motor 5/5 throughout , DTR 2+ bilaterally

Data interpretation : Based on what you have learned form the history and physical examination, list up to 3 diagnosis that might explain this patient¡¦s complaint(s). List your diagnosis form most to least likely. For some cases , fewer than 3 diagnosis will be appropriate . then , enter the positive or negative findings form the history and the physical examination (if present) that support each diagnosis. Lastly , list initial diagnostic studies(if any) you would order for each listed diagnosis (e.g., restricted physical exam maneuvers , laboratory tests, imaging, ECG, etc.).

Diagnosis #2 : Depression

history finding(s)
. Fatigue . Weight loss
. Death of spouse
. Terminal insomnia

Physical Exam finding(s)
. None

Diagnosis #1 : GI malignancy (eg , colon cancer, Gastric cancer )

History finding(s)
. Fatigue
. Weight loss
. Early satiety

Physical exam finding(s)
. None

Diagnosis #3 : hyperthyroidism

History finding(s)
. Weight loss
. Fatigue

Physical Exam finding(s)
. None

Diagnosis Studies
. TSH and T4
. CBC with differential
. Rectal examination with FOBT
. Colonoscopy
. CT scan ion the abdomen







µoªí¤å³¹®É¶¡2018/08/15 03:22pm¡@IP: ¤w³]©w«O±K[¥»¤å¦@ 8614 ¦ì¤¸²Õ]¡@ 
JuanFe 

 

¸ê®Æ: ¦¹·|­û¥Ø«e¤£¦b½u¤W Male ¨°Às ¥Õ¦Ï®y
«Â±æ: 0
¾y¤O: ¾y¤O: 78912
¸gÅç: ¸gÅç: 35968
¨Ó¦Û: ¦t©z¤¤¡@blank
µo¤å: 1118 ½g
ºëµØ: 0 ½g
¦b½u: 47¤Ñ19®É11¤À43¬í
µù¥U: 2013/06/17
Message ¬d¬Ý¡@·j´M¡@³q°T¿ý¡@½Æ»s¡@¤Þ¥Î¡@¦^ÂФ峹¦^ÂС@[²Ä 8 ¼Ó]
 7 Case 7 & scenario (diarrhea)

Doorway information about patient

The patient is a 35-year-old man who comes to the office due to acute diarrhea.

Vital signs
. Temperature: 36.7¡¦C(98.1F)
. Blood pressure: 110/65 mmHg
. Pulse : 100 /min
. Respirations: 18/min

Basi differential diagnosis

. Viral gastroenteritis
. Bacterial gastroenteritis & food-borne pathogens
. Medication induced
. Giardiasis
. Clostridium difficle colitis
. Inflammatory bowel disease
. Irritable bowel disease
. Malabsorption . HIV

¡X¡X¡X¡X¡X

Case 7 sim. pt. instructions

If the doctor asks you about anything otters than these . just say ¡§no¡¨ , or provide an answer that a normal patient might give.

You are a 35-year-old man who has diarrhea.

History of present illness

. Onset 1 day ago , 6-8 hours after meal
. Ate salad and shrimp at a local seafood restaurant
. Loose , watery stop,s with no blood or mucus
. 6 or 7 bowel movements in 24 hours
. Associated symptoms:
- Diffuse abdominal cramps
- Nausea and vomiting
- Difficulty keeping down solids or liquids
- no fever or chills
. Nothing seems to make it worse , and you have not tried any over-the-counter treatments
. 2 friends who ate with you have similar symptoms
. No recent travel

Review of system

. Sinus infection 2 weeks ago , treated with amoxicillin (last dose 2 days ago)
. No chest pain or shortness of breath
. No urinary symptoms
. No back pain

Past medical / family / social history

. No significant past medical problems or surgeries
. No other medications (otters than amoxicillin)
. No medication allergies
. Bother parents and siblings (2 brothers) are healthy
. Single , live  alone
. Occupation : Software engineer
. Smoking : No
. Alcohol : no
. Recreational drugs: No

Physical examination

Head and neck:
. No erythema or exudates in the mouth / pharynx
. Dry mucous membranes
. No enlarged lymph nodes

Skin:
. No jaundice

Chest/lungs:
. Clear to auscultation bilaterally

Heart : . regular rhythm without murmurs , gallops , or rubs

Abdomen:
. Non tender, non distended
. Normative bowel sounds throughout
. Tympanic to percussion
. No hepatosplenomegaly

¡X¡X¡X¡X¡X

Case 7 sim. pt. checklist

Following the encounter , check which of the following items were performed by the examinee

History of present illness / review of systems

. Asked to clarify characteristics of diarrhea
- Increased frequency ? Increased volume ? Altered consistency ? . Asked about the frequency of diarrhea
. Asked about associated symptoms (eg,vomiting , fever , abdominal pain, anorexia, prior constipation , myalgia , tenesmus)
. Asked about any blood or mucus in stools
. Asked about any recent travel
. Asked whether any otters family members or other contacts are sick
. Asked about exposure to suspicious foods (eg, unpasteurized /undercooked food , unusual foods , dairy products , seafood)

Past medical / family/ social history

. Asked about prior episodes of diarrhea and gastrointestinal illness
. Asked about otters medical issue
. Asked  bout medications (especially antibiotics) and medication allergies
. Asked  about recent and previous hospitalizations
. Asked boy any  abdomen surgeries
. Asked bout occupation
. Asked boy tobacco , alcohol , and drug use
. Asked about family history (especially gastrointestinal disease)

Examination

. Examinee washed heads
. Examined without gown , not though gown
. Auscultated abdomen
. Palpated abdomen superficially
. Palpated abdomen deeply
. Examination of skin for any rashes
. Examination of oral cavity
. Respiratory examination
. Cardiac auscultation

Counseling

. Explained the physical findings and possible diagnosis
. Explained further workup
. Discussed need for rectal examination
. Discussed fluids and otters basic interventions

Communication skills and professional conduct

. Knocked before entering the room
. Introduced self and greeted you warmly
. Used your name to address you
. Paid attention to heat you said and maintained good eye contact
. Asked open-ended questions
. Asked non-leading questions
. Asked one question at a time
. Listened to what you said without interrupting
. Used plain English rather than technical jargon
. Used appropriate transition sentences
. Used appropriate draping techniques
. Summarized the history and explained physical findings
. Asked whether you have any concerns / questions

Differential diagnosis

.Viral gastroenteritis
. Bacterial gastroenteritis
. Clostridium difficile diarrhea

Diagnostic study/ studies

. Rectal examination
. Fecal occult blood test
. CBC with differential count
. Basic metabolic panel (electrolytes , BUN , creatinine, glucose)
. Stool for C. difficile toxin
. Stool for fecal leukocytes

¡X¡X¡X¡X¡X

Case 7 clinical summary

Clinical Skills Evaluation
Case 7 Patient Note

The following represents a typical note for this patient encounter . the details amy vary depending on the information given by the simulated patient.

history : Describe the history you jus obtained for  this patient . Include only information (permanent positives and negatives) relevant to this patient¡¦s problem(s)

. 35-yo male with 1 day of diarrhea with 6-7 loose BM/day without blood or mucus.
. Ate seafood and salad at restaurant 6-8 hours before symptom onset.
. Recent sinus infection treated with amoxicillin , last dose 2 days ago.
. Diffuse crampy abdominal pain , nausea , vomiting , decreased PO intake. , Two otters friends who ate at the restaurant have the same symptoms,

ROS : No fever , chills , chest pain , shortness of breath , burning with urination , rennet travel , or back pain.
PMHx : None
PSHx : None
Meds : None
Allergies : None
FHx : Noncontributory
SHx : Denies tobacco and alcohol use

Physical examination ; Describe any positive and negative findings relevant to this patient¡¦s problem(s). Be careful to include only those parts of the examination performed in this encounter.

. Viral signs: Temperature , 36.7¡¦C (98F) ; blood pressure , 110/65 mmHg : pulse , 100/min ; and respirations , 18/min.
. Head / neck : Oropharynx with dry mucous membranes but no erythema or exudates , no enlarged lymph nodes , no jaundice
. Chest /lungs : Clear to auscultation bilaterally
. Heart : RRR without M/G/R
. Abdomen : Contender , non distended , normoactive bowel sounds thought ; tympanic on percussion ; no hepatosplenomegaly

Data interpretation : Based on what you have learned form the history and physical examination , List up to 3 diagnosis that might explain this patient¡¦s complaint(s). List your diagnosis form most to least likely . For some cases , fewer than 3 diagnosis will be appropriate . Then , enter the positive or negative findings forth history and the physical examination (if present) that support each diagnosis . Lastly , list initial diagnostic studies (if any) you would order for each listed diagnosis (e,g,,restricted physical exam maneuvers , laboratory test, imaging , ECG , etc.).

Diagnosis #1 : Viral gastroenteritis

History finding(s)
. Diarrhea without fever
. No blood in stool
. Other sick friends who also ate at same place

Physical Exam finding(s)
. No fever

Diagnosis #2 : Bacterial gastroenteritis

History finding(s)
. Symptom onset 6-8 hours after eating at a restaurant
. Nausea , vomiting

Physical Exam finding(s)
. None

Diagnosis #3 : Clostridium difficile colitis

History finding(s)
. Symptoms starting 2 days after rennet amoxicillin use
. No blood in stool

Physical Exam finding(s)
. None

Diagnostic Studies
. Stool for leukocytes
. Stool for Clostridium difficile
. Rectal examination with FOBT







µoªí¤å³¹®É¶¡2018/08/15 03:23pm¡@IP: ¤w³]©w«O±K[¥»¤å¦@ 8052 ¦ì¤¸²Õ]¡@ 
JuanFe 

 

¸ê®Æ: ¦¹·|­û¥Ø«e¤£¦b½u¤W Male ¨°Às ¥Õ¦Ï®y
«Â±æ: 0
¾y¤O: ¾y¤O: 78912
¸gÅç: ¸gÅç: 35968
¨Ó¦Û: ¦t©z¤¤¡@blank
µo¤å: 1118 ½g
ºëµØ: 0 ½g
¦b½u: 47¤Ñ19®É11¤À43¬í
µù¥U: 2013/06/17
Message ¬d¬Ý¡@·j´M¡@³q°T¿ý¡@½Æ»s¡@¤Þ¥Î¡@¦^ÂФ峹¦^ÂС@[²Ä 9 ¼Ó]
 8 Case 8 scenario (sore throat)

Doorway information about patient

The patient is a 25 -year-old woman who comes to the clinic sue to a sore throat

Vital signs
. Temperature : 13.3¡¦C (101F)
. Blood pressure : 120/70 mmHg
. Pulse : 90 /min
. Respirations : 22/min

Basic differential diagnosis
. Viral pharyngitis
. Bacterial pharyngitis
- Group A streptococcal pharyngitis
- Mycoplasma pneumonia
- Neisseria gonorrhea
. Epstein-Barr virus mononucleosis
. Allergies rhinitis / postnasal drip
. Chronic tonsillitis
. Primary HIV

¡X¡X¡X¡X¡X

Case 8 cim. pt. instructions

if the doctor sales you about anything other than these ,just say ¡§no,¡¨ or provide an answer that a normal patient might give.

you are a 25 -year-old woman who comes to the office with a sore throat

History  of present illness

. Onset 3 days ago
. Pain with swallowing food
. Associated symptoms:
- Headache
- Body ache
- Nasal congestion
- Dry cough
- Fever to 38.1¡¦C(100.5F)
. Symptoms partially relieved with Tylenol/acetaminophen
. Boyfriend had similar symptoms 2 weeks ago but now better

Review of system

. No abdominal pain , pelvic pain , rash , chills , chest pain , or shortness of breath
. Last menstrual period was 2 weeks ago

Past medical / family/ social history

. Frequent episodes of tonsillitis in childhood
. 2 upper respiratory illnesses in the past 2 years
. No pregnancies
. No past surgeries
. No current medications
. No medication allergies
. Parents and sister are healthy
. Occupation : College student
. Sexually active with boyfriend (condoms for birth control)
. Smoking : No
. Alcohol : No
. Recreational drugs : No

Physical examination

Head and neck :
. Oropharynx with tonsillar exudates
. Tympanic membranes clear bilaterally
. No sinus tenderness
. No enlarged lymph nodes

Chest/lungs: . Clear to auscultation  bilaterally

Heart : . Regular rhythm without murmurs, gallops , or rubs

Abdomen ; . Non tender , non distended
. Normative bowel sounds throughout
. Tympanic to percussion
. No hepatospenomegaly

¡X¡X¡X¡X¡X

Case 8 sim. pt. check list

Following the encounter , check which of the following items were performed by the examinee

History of present illness / review or system

. Asked about the onset of sore throat
. Asked about the course of symptoms over time
. Asked about pain during swallowing
. Asked  about associated symptoms
- Coughing and breathing problems
- Nasal discharge/ congestion
- Sinus pain and postnasal drip
- Headache
- Fever and chills
- Nausea and vomiting
- Joint pains and muscle aches
- Swollen neck glands
- Abdominal pain (especially left upper quadrant and pelvic pain)
- Rash
- Vaginal discharge
. Asked about contacts with ill person

Past medical /family/social history

. Asked about similar episodes in the past
. Asked about otters past medical issues
. Asked about current medications
. Asked about medication allergies
. Asked about family health problems
. Asked  about occupation
. Asked about tobacco , alcohol , and recreational drug use
. Asked about sexual and reproductive history

Examination

. Washed hands before examination
. Examined without gown , not though gown
. Looked inside mouth
. Palpated cervical lymph nodes
. Examined both ears
. Palpated spleen and liver
. Palpated abdomen (superficial and deep)
. Auscultated heart
. Auscultated lungs
. Examined skin for rash

Counseling

. Explained physical findings and possible diagnosis
. explained further workup

Communication skills and professional conduct

. Knocked before entering the room
. Introduced self and greeted you warmly
. Used your name to address you
. Paid attention to what you said and maintained good eye contact
. Asked open-ended questions
. Asked non;eating questions
. Asked one question at a time
. Listened to what you said without interrupting
. Used plain English rather than technical jargon
. Used appropriate transition sentences
. Used appropriate draping techniques
. Summarized the history and explained physical findings
. Expressed empathy and gave appropriate reassurances
. Asked whether you have any concerns/questions

Differential diagnosis

. Infectious mononucleosis
. Viral pharyngitis
. Bacterial (streptococcal) pharyngitis

Diagnostic study/workup

. CBC
. Monospot test
. Rapid streptococcal antigen test

¡X¡X¡X¡X¡X

Case 8 clinical summary

Clinical Skills Evaluation
Case 8 Patient Note

The following represents a typical note for this patient encounter . the details amy vary depending on the information given by the simulated patient.

History : Describe the history you just obtained form this patient . include only information (pertinent positives and negatives) relevant to this patient¡¦s problem(s).

. 25-yo female with 3 days pop sore throat.
. Initially , sore throat , sinus congestion , runny nose , fever , and dry cough
. Sore throat has worsened with pain when swallowing.
. Sick contact (boyfriend) had similar symptoms that resolved

ROS : No chills , chest pain ,shortness of breath , nausea , vomiting m or abdominal pain
PMHx : None
PSHx : None
Meds : None
Allergies : None
FHx : Noncontributory
SHx : Denies tobacco and alcohol use

Physical examination: Describe any positive and negative findings relevant to this patient¡¦s problem(s) . be careful to include only those parts of the examination performed in this encounter

. Vital signs : Temperature , 38.3¡¦C (101F); blood pressure , 120/70 mmHg; pulse , 90/min ; and respirations , 16/min
. HEENT : Oropharynx with tonsillar exudates , TM clear bilaterally
. Neck : no enlarged lymph nodes
. Chest / lungs ; clear to auscultation bilaterally
. Heart : RRR without M/R/G
. Abdomen : non tender , non distended , normative bowel sounds thought ; tympanic to percussion ; no hepatosplenomegaly.

Data interpretation : based on what you have learned form the history and physical examination, list up to 3 diagnosis that might explain this patient¡¦s complaint(s) . List your diagnosis form most to least likely . for some cases , fewer than 3 diagnosis will be appropriate . then , enter the positive or negative findings form the history and the physical examination (if present) that support each diagnosis . Lastly , list initial diagnostic studies (if any) you would order for each listed diagnosis (e,g,, restricted physical exam maneuvers , laboratory tests , imaging , ECG , etc.).

Diagnosis #1 : Bacterial pharyngitis

History finding(s)
. Sore throat
. Fever

Physical examination finding(s)
. Fever
. Tonsillar exudates

Diagnosis #2 : Viral pharyngitis

History finding(s)
. Fever
. Sore throat
. Boyfriend with similar illness that resolved

Physical examination finding(s)
. Fever

Diagnostic studies
. Rapid strep test
. Throat culture if rapid strep test is negative







µoªí¤å³¹®É¶¡2018/08/15 03:25pm¡@IP: ¤w³]©w«O±K[¥»¤å¦@ 7224 ¦ì¤¸²Õ]¡@ 
JuanFe 

 

¸ê®Æ: ¦¹·|­û¥Ø«e¤£¦b½u¤W Male ¨°Às ¥Õ¦Ï®y
«Â±æ: 0
¾y¤O: ¾y¤O: 78912
¸gÅç: ¸gÅç: 35968
¨Ó¦Û: ¦t©z¤¤¡@blank
µo¤å: 1118 ½g
ºëµØ: 0 ½g
¦b½u: 47¤Ñ19®É11¤À43¬í
µù¥U: 2013/06/17
Message ¬d¬Ý¡@·j´M¡@³q°T¿ý¡@½Æ»s¡@¤Þ¥Î¡@¦^ÂФ峹¦^ÂС@[²Ä 10 ¼Ó]
 9 Case 9 scenario (medication refill)

Doorway information about patient

The patient is a 56-year-old man who comes to the office for a blood pressure check and medication refill

Vital signs
. Temperature : 36.1¡¦C (97F)
. Blood pressure ; 150/90 mmHg
. Pulse : 80 /min
. Respirations : 16/min

Basic differential diagnosis

When patient come for follow-up of known conditions , the primary diagnosis is known . However, you should be aware of complications and associated conditions.

. Congestive cardiac failure
. Coronary artery disease (angina)
. Peripheral vascular disease
. Retinopathy
. Side effects of the medications

¡X¡X¡X¡X¡X

Case 9 sim. pt. instruction

If the doctor asks you about anything other than these , just say ¡§ no,¡¨ or provide an answer that a normal patient might give.

You are a 56-year - old man who needs a blood pressure recheck and medication refill.

history of present illness

. Diagnosed with hypertension and high cholesterol 10 years ago
. Taking medications regularly
. Currently medications include hydrochlorothiazide and simvastatin
. DO not volunteer this information unless asked specifically abut past treatment:
You were initially treated with lisinopril , but that was stopped due to a dry cough ; You were then treated with atenolol but developed erectile dysfunction
. Not exercising regularly and not always following dietary recommendations
. Checking blood pressure at home ranges form 140-150 systolic and 80-90 diastolic
. No headaches , palpations , blurry vision , chest pain , shortness of breath , nosebleeds , dizziness , or leg swelling.

Past medical / family / social history

. Medical history otherwise negative
. No surgeries
. No other medications
. No medication allergies
. Father has hypertension and  mother has diabetes
. Occupation : Accountant
. Smoking ; 1 pack /day for 30 years
. Alcohol ; 1 glass of wine on social occasions
. Recreational drugs : None

Physical examination

HEENT:
. Pupils are equally round and reactive to light and accommodation (PERRLA)
. Extra ocular movement are intact (EOMI)
. Funduscopic examination shows no papilledema , exudates or AV nicking

Neck:
. No enlarged lymph nodes
. No bruits
. Carotid pulse 2+ bilaterally

Chest/lungs: . Clear to auscultation bilaterally

Heart : . Regular rhythm without murmurs , gallops , or rubs

Extremities :
. No cyanosis , clubbing , or edema
. Radial and posterior tibial pulses 2+

¡X¡X¡X¡X¡X

Case 9 sim. pt. checklist

Following the encounter , check which of the following items were performed by the examinee

History of present illness/review of systems

. Asked when you diagnosed with high blood pressure
. Asked what medications you are taking and of you take them as prescribed
. Asked about any side effects or other issues with medications . Asked about prior treatment and why it was changed
. Asked about symptoms that might indicate a problem , including:
- Headache
- Dizziness
- Nosebleeds
- Dyspnea , orthopnea, paroxysmal nocturnal dyspnea
- Palpations
- Chest pian
- Pedal edema
. Asked about home blood pressure checks
. Asked about diet and exercise habits

Past medical / family / social history

. Asked about otters medical issues , hospitalizations , and surgeries
. Asked about other medications . Asked about medication allergies
. Asked about family health (especially cardiovascular daises)
. Asked about tobacco . alcohol , and recreational drugs use
. Asked about occupation

Examination

. Washed hands before examination
. Examined without gown , not through gown . Measured blood pressure in both arms
. Examined eyes with ophthalmoscope
. Check carotid arteries (pulses and bruits)
. Checked jugular venous pressure
. Auscultated heart
. Auscultated lungs
. Palpated peripheral pulses

Counseling

. Complemented you for using medications as prescribed
. Complemented you for checking home blood pressure regularly
. Explained further workup
. Explained the importance of diet and regular exercise
. Explained likely complications of uncontrolled blood pressure

Communication skills and professional conduct

. Knocked beef entering the room
. Introduced self and greeted you warmly
. Used your name to address you
. Paid attention to what you said and maintain good eye contact
. Asked open-ended questions
. Asked non-leading questions
. Asked one question at a time
. Listened to what you said without interrupting
. Used plain English rather than technical jargon
. Used appropriate transition sentences
. Used appropriate draping techniques
. Summarized the history and explained physical findings
. Expressed empathy and gave appropriate reassurances
. Asked whether you have any concerns/questions

Differential diagnosis

. Essential hypertension

Diagnostic study/studies

. Urinalysis
. Lipid profile
. Electrolytes
. BUN and creatinine
. Glucose
. ECG

¡X¡X¡X¡X¡X

Case 9 clinical summary

Clinical Skill evaluation
Case 9 Patient Notes

The following represent a typical notes for this patient encounter . the detail may vary depending on the information given by the simulated patient.

History : Describe the history you just obtained form this patient. Include only information (pertinent positives and negatives ) relevant to this patient¡¦s problems(s)

. 56-yo man here for BP check and medical refill.
. BP ranges form 140-150 systolic and 80-90 diastolic at home 3x/week in morning and evening
. Not compliant with low salt diet , no regular exercise
. Compliant with medications.
. Cough with ACE inhibitor , ED with beta blocker

ROS : no headache , palpations, blurry vision , chest pain , shortness of breath , nose bleeds, dizziness , or leg swelling
PMHx : Hypertension and hypercholesterolemia for past 10 years .
PSHx: None
Meds : Hydrochlorothiazide 50 mg daily , simvastatin 20 mg daily
Allergies ; None
FHx ; father has hypertension , mother has diabetes
SHx: Smokes 1 PPD for 30 years . drinks occasionally

Physical examination : Describe nay positive and negative findings relevant to this patient¡¦s problem(s) . be careful to include only those parts of the examination performed in this encounter.

. Vital signs : Temperature , 37¡¦C (98.6F) ; blood pressure , 150/90 mmHg in both arms ; pulse , 90/min; and respirations , 16/min
. HEENT : PERRLA , EOMI , fund without papilledema , exudates , or AV nicking
. Neck : No enlarged lymph nodes , no bruits
. Chest /lungs : Clear to auscultation bilaterally
. Heart : RRR without M/G/R
. Extremities : No cyanosis , clubbing , or edema ; pulse 2+ bilaterally in carotid , radial , and posterior tibialis

Data interpretation :
based on what you have learned from the history and physical examination , list up to 3 diagnoses that might explain this patient¡¦s compliant(s) .List your diagnosis form most to least likely . For  some cases , fewer than 3 diagnosis will be appropriate . then enter the positive or negative findings form the history and the physical examination (if present) that support each diagnosis . Lastly , list initial diagnostic studies (if any ) you would order for each listed diagnosis (eg , restricted physical examination maneuvers , laborite tests , imaging ECG)

Diagnosis #1 : essential hypertension , sub optimally controlled

history finding(s)
. Hypertension
. Poor dietary complicate
. No regular exercise

Physical examination finding(s)
. Elevated blood pressure to 150/90 mmHg
. No neck bruits
. Symmetrical peripheral pulses
. Normla funduscopic examination

Diagnostic studies
. Serum electrolytes and creatinine
. Lipid panel







µoªí¤å³¹®É¶¡2018/08/15 03:26pm¡@IP: ¤w³]©w«O±K[¥»¤å¦@ 7971 ¦ì¤¸²Õ]¡@ 
JuanFe 

 

¸ê®Æ: ¦¹·|­û¥Ø«e¤£¦b½u¤W Male ¨°Às ¥Õ¦Ï®y
«Â±æ: 0
¾y¤O: ¾y¤O: 78912
¸gÅç: ¸gÅç: 35968
¨Ó¦Û: ¦t©z¤¤¡@blank
µo¤å: 1118 ½g
ºëµØ: 0 ½g
¦b½u: 47¤Ñ19®É11¤À43¬í
µù¥U: 2013/06/17
Message ¬d¬Ý¡@·j´M¡@³q°T¿ý¡@½Æ»s¡@¤Þ¥Î¡@¦^ÂФ峹¦^ÂС@[²Ä 11 ¼Ó]
 10 Case 10 scenario (constipation)

Doorway information about patient

The paint is a 66-year-old man who comes to the clinic due to constipation

Vital signs
. Temperature ; 36.7¡¦C (98F)
. Blood pressure : 120/70 mmHg
. Pulse : 70 /min
. Respirations : 16/min

Basic differential diagnosis

. Functional constipation
. Obstructive lesions (eg , bowel obstruction , carcinoma f colon)
. Metabolic disturbances (eg , hypothyroidism, diabetes mellitus , hypercalcemia)
. Neurologic dysfunction (eg , stroke , autonomic neuropathy , final cord trauma , multiple sclerosis, Parkinson disease)
. Medication-induced (eg , iron preparations , opiates , anticholinergics)

¡X¡X¡X¡X¡X

Case 10 sim. pt. instructions

If the doctor asks you about anything other than ashes , just say ¡§no,¡¨ or provide na answer that a normal patient might give.

You are a 66-year -old man with constipation.

history o present illness

. Onset 5 months ago
. Worse over the last 2 months
. Previously 1 film bowel movement a day , now every otters day
. Stool now become harder
. Straining , difficulty passing stool , sense of incomplete evacuation
. Occasional black stools; no red blood in stools
. eating fruits and vegetables regularly and have not changed diet recently

Review of systems

. Fatigue
. 10-lb weight loss over the last 2 months
. No fever , chills , night sweats
. No diarrhea , nausea , vomiting
. No urinary symptoms

Past medical / family / social history

. Arthritis of the right knee
. Hashimoto thyroiditis ; had normal blood work 1 year ago
. Never had colonoscopy but rectal examination was normal 2 years ago
. No surgeries
. Medications:
- Levothyroxine 100 mcg/day
- Hydrocodone/acetaminophen 5mg/650mg 1 pill 3 times /day 9started 2 months ago for knee pain
. No medication allergies
. After died of colon cancer at age 67 and mother is healthy
. Occupation; Supervisor at pharmaceutical company
. Tobacco : No
. Alcohol : 2-3 glasses of wine a week
. Recreational drug : No

Physical examination

HEENT :
. No pallor icterus

Neck :
. No enlarges lymph nodes

Chest /lungs :
. Clear to auscultation bilaterally

Heart : . Regular rhythm without murmurs , gallops , or rubs

Abdomen :
. Non-tender, non-distended
. Hypoactive bowel sounds thought
. No hepatosplenomegaly

Neurologic :
. Muscle strength 5/5 throughout
. Reflexes 2+ symmetric

¡X¡X¡X¡X¡X

Case 10 sim. pt. checklist

Following the encounter , check which for the following itms were performed by the examinee.

history of present illness / review of systems

. Asked an open-ended questions What do you mean by constipation)
. Asked about the onset of constipation
. Asked about the frequency of bowel movements
. Asked about amount and caliber of stool passed
. Asked about consistency of stool
. Asked about pain during defecation
. Asked about any blood in stools of black stools
. Asked about episodes of diarrhea
. Asked about nay nausea and vomiting
. Asked about abdominal pain or cramps
. Asked about urinary issue (polyuria, dribbling)
. Asked about intolerance to hot or cold temperatures
. Asked about loss of appetite and weight loss
. Asked about diet (especially fluids and dietary fiber)

Past medical / family /social history

. Asked about similar episodes in the past
. Asked about other medical issues
. Asked about previous hospitalizations and surgeries
. Asked about regular screening procedures (especially colon conner screening)
. Asked about current medications
. Asked about medication allergies
. Asked about family health (especially colon cancer)
. Asked about tobacco , alcohol , and recreational rug use
. Asked about occupation

Examination

. Washed hands before examination
. Examined without gown , not though gown
. Examined eyes for pallor
. Auscultated abdomen
. Palpated abdomen (superficial and deep)
. Checked muscle power and reflexes inlayer extremities

Counseling

. Explained the physical findings and possible diagnosis
. Explained further workup (include rectal examination)

Communication skills and professional conduct

. Knocked before entering the room
. introduced self and greeted you warmly
. Used your name to address you
. Paid attention to what you said and maintained good eye contact
. Asked opened questions
. Asked non-leading questions . Asked one question at a time
. Listened to what you said without interrupting me
. Used plain English rather than technical jargon
. Used appropriate transition sentences
. Used appropriate draping techniques
. Summarized the history and explained physical findings
. Expressed empathy and gave appropriate reassurances
. Asked whether you have any concerns/questions

Differential diagnosis

. Medication -induced
. Carcinoma of colon
. Uncontrolled hypothyroidism
. Functional constipation
. Undiagnosed diabetes

Diagnostic study/studies

. Rectal examination and stools for occult blood
. CBC with differential . TSH
. Fasting blood sugar and /or hemoglobin A1c
. Colonoscopy

¡X¡X¡X¡X¡X

Case 10 clinical summary

Clinical Skills Evaluation
Case 10 Patient notes

The following represents a typical note for this patient encounter . the details may vary depending on the information given by the simulated patient

History : describe the history you just obtained form this patient . Include only information (pertinent positives and negatives) relevant to this patient¡¦s problem(s).

. 66-yo man here for constipation for 5 months with worsening over past 2 months
. Fatigue , 4.5-kg (10-lb) weight loss
. Change in bowel movement form 1 /day to 1 every other day
. Occasional black stools

ROS : No diarrhea , abdominal pain , nausea , vomiting , fever , chills , night sweats , or urinary problems
PMHx: Hashimoto¡¦s thyroiditis , severe DJD of right knee
PSHx: None
Meds : Levothyroxine , hydrocodone / acetaminophen (started 2 months ago)
Allergies : None
FHx : Father died at age 67 of colon cancer
SHx : no smoking but drinks 1-3 glasses of wine /week

Physical examinations :¡¨ Describe any positive and negative findings relevant to this patient¡¦s problem(s). Be careful to include only those parts of the examination preformed in this encounter.

. Vital signs: Temperature, 36.7¡¦C(98F); blood pressure , 120/70 mmHgin both arms; pulse , 70/min; respirations , 16/min
. HEENT : No pallor or jaundice
. Neck : No enlarge lymph nodes
. Chest / lungs : clear to auscultation bilaterally
. Heart : Regular rate and rhythm without M/G/R
. Abdomen : Non-tender , non-distended , hypoactive bowel sounds thought , no hepatosplenomegaly
. Neurologic : Muscle strength 5/5 throughout , DTR 2 + bilaterally and symmetrical

Data interpretation : Based on what you have learned from the history and physical examination, list up to 3 diagnosis that might explain this patient¡¦s complaint(s) . List your diagnoses from most to least likely . For some cases , fewer than 3 diagnosis will be appropriate . the , enter the positive and negative findings form the history and the physical examination (if present) that support each diagnosis . Finally , list initial diagnostic (if any) you would order for each listed diagnosis (eg,restricted physical examination maneuvers, laboratory tests , imaging ECG , etc).

Diagnosis #1 : Colon cancer

history findings(s)

. Worsening constipation
. Fatigue
. Weight loss
. Black stools

Physical examination finding(s)

. hypoactive bowel sounds

Diagnosis #2 : Functional constipation form medications

History finding(s)

. New drug (Lortab) started , with worsening of constipation

physical examination finding(s)

. Hypoactive bowel sounds

Diagnosis #3 : Hypothyroidism with suboptimal control

History finding(s)

. Fatigue . Constipation

Diagnostic studies

. Rectal examination with stool for occult blood
. CBC with differential
. Serum TSH
. Colonoscopy
. Serum calcium







µoªí¤å³¹®É¶¡2018/08/15 03:27pm¡@IP: ¤w³]©w«O±K[¥»¤å¦@ 8249 ¦ì¤¸²Õ]¡@ 
JuanFe 

 

¸ê®Æ: ¦¹·|­û¥Ø«e¤£¦b½u¤W Male ¨°Às ¥Õ¦Ï®y
«Â±æ: 0
¾y¤O: ¾y¤O: 78912
¸gÅç: ¸gÅç: 35968
¨Ó¦Û: ¦t©z¤¤¡@blank
µo¤å: 1118 ½g
ºëµØ: 0 ½g
¦b½u: 47¤Ñ19®É11¤À43¬í
µù¥U: 2013/06/17
Message ¬d¬Ý¡@·j´M¡@³q°T¿ý¡@½Æ»s¡@¤Þ¥Î¡@¦^ÂФ峹¦^ÂС@[²Ä 12 ¼Ó]
 11 Case 11 scenario (importance)

Doorway information about patient

The patient is a 50-year-old man who comes to the clinic due to importance

Vital signs . Temperature : 36.7¡¦C (98F)
. Blood pressure : 150/80 mmHg
. Pulse : 80/min
. respirations ; 16/min

Basic differential diagnosis Cardiovascular
. Atherosclerotic vascular disease

Metabolic /endocrine
. Diabetes
. Hypogonadism
. Hyperprolactinemia

Neurotic
. Spinal cord disorders

Psychological
. Anxiety
. Depression
. Alcohol or otters substance abuse

Other
. Medications (eg, antihypertensives)

¡X¡X¡X¡X¡X

Case 11 sim.pt. instruction

If the doctor asks you about anything other than these , just say ¡§ no ,¡¨ or provide an answer that a normal patient might give.

You are a 50-year-old man who has erectile dysfunction

History of present illness

. Onset 3-4 months ago
. Gradually increasing difficulty getting an erection
. Normal interest unisexual activity
. Increased stress over last 6 months due to financial problems

Do not volunteer this information unless asked : You have awakened with a nocturnal erection for several months.

Review of systems . Mild fatigue
. No headaches or visual changes
. No pain in the extremities
. No nausea , vomiting , or abdomen pain

Past medical history

. Diabetes for 10 years (home glucose ranges 150-200 mg/dL)
. Hypertension
. Generalized anxiety disorder
. Surgeries : None
. medications : Atenolol 50 mg , daily (started 4 months ago), lisinporil 20 mg daily, metformin 500 mg twice daily , glyburide 10 mg daily, fluoxetine 20 mg daily
. Allergies : None
. Immediate family members are healthy
. Occupation : truck driver
. Married, live with wife
. Tobacco 1-2 cigarettes week 9only when gong out with friends)
. Alcohol : 2-3 beers a day for 25 years
. Recreational drugs : No

Physical examination

HEENT:

. PERRLA
. EMOI

Abdomen

. Non-tender, Non-distended
. Normative bowel sounds throughout
. No hepatosplenomegaly
. No bruits

Extremities

. Posterior tibial and dorsals pedis pulse 2+ in both lower extremities

¡X¡X¡X¡X¡X

Case 11 sim. pt. checklist

Following the encounter , check which of the following items were performed by the examinee

History of present illness/ review of systems

. Asked about the onset of symptoms
. Asked whether out is continuous or intermittent
. Asked whether it is getting worse
. Asked about any changes in sexual desire
. Asked about any problems with ejaculation
. Asked detailed sexual history including the number of sexual partners (if multiple , ask follow-up questions : Dose the dysfunction occur with one partner and not another?)
. Asked about nocturnal erections
. Asked about aggravating to triggering factors
. Asked about nay pain in the legs(claudication)
. Asked about anxiety and depression
. Asked about headache (pituitary tumors)
. Asked about trauma

Past medical /family/social history

. Asked about otters medical issue (especially hypertension , diabetes mellitus , sickle cell disease , pulmonary vascular disease),hospitalization , and surgeries
. Asked about current medications
. Asked about medication allergies
. asked about family health
. Asked about tobacco, alcohol, and drug use
. Asked about occupation

Examination

. Washed hands before examination
. Examined without gown , not though gown
. Palpated abdomen and listened for bruit
. Examined pulsations in lower limbs

Counseling

. Explained the physical findings aden possible diagnosis
. Explained the need for additional workup(include genitourinary examination)
. Discussed quoting smoking and reducing alcohol intake

Communication skills and professional conduct

. Knocked before entering the room
. introduced self and greeted you warmly
. Used your name to address you
. Paid attention to what you said and maintained good eye contact
. Asked opened questions
. Asked non-leading questions . Asked one question at a time
. Listened to what you said without interrupting me
. Used plain English rather than technical jargon
. Used appropriate transition sentences
. Used appropriate draping techniques
. Summarized the history and explained physical findings
. Expressed empathy and gave appropriate reassurances
. Asked whether you have any concerns/questions

Differential diagnosis

. Medication induced
. Diabetes neuropathy
. Atherosclerotic vascular disease
. Anxiety

Diagnostic study/studies

. Genital examination
. Fasting blood sugar and hemoglobin A1c
. Complete blood count
. TSH , Serum prolactin , and testosterone

¡X¡X¡X¡X¡X

Case 11 clinical summary

Clinical Skills Evaluation
Case 11 Patient Note

The following represents a typical note for this patient encounter . the detail may vary depending on the information given by the simulated patient.

History : Describe the history you just obtained form this patient . Include only information (Pertinent positives and negatives)  relevant to this patient¡¦s problem(s).

. 50-yo man with 4 months of worsening erectile dysfunction.
. New blood pressure medication (atenolol) started 4 months ago.
. Increased stress for past 6 months , mild fatigue.
. Poorly controlled diabetes with glucose near 200 mg/dl most of the day.

ROS : No headaches , leg pain , visual disturbances , nausea , vomiting , or abdominal pain
PNHx: Diabetes ,hypertension, anxiety
Meds ; Metformin, glyburide, fluoxetine , atenolol (started 4 months ago)
Allergies : None
FHx : Parents and siblings are healthy
SHx : Occasionally smokes and has had 2 or 3 beers/day for 25 years

Physical examinations : Describe any positive and  negative findings relevant  to this patient¡¦s problem(s) . be careful to include only those parts of the examination performed in this encounter.

. Vital signs : Temperature, 36.7¡¦C (98F) ; blood pressure , 150/80 mmHg ; pulse , 80/min ; respirations , 16/min
. HEENT : PERRLA, EOMI
. Abdomen : Non-tender , non-distended, normative bowel sounds throughout , no hepatosplenomegaly or bruits
. Extremities : Pulses 2+ in the bilateral lower extremities

Data interpretation : Based on what you have learned from the history and physical examination, list up to 3 diagnosis that might explain this patient¡¦s complaint(s) . List your diagnoses from most to least likely . For some cases , fewer than 3 diagnosis will be appropriate . the , enter the positive and negative findings form the history and the physical examination (if present) that support each diagnosis . Finally , list initial diagnostic (if any) you would order for each listed diagnosis (eg,restricted physical examination maneuvers, laboratory tests , imaging ECG , etc).

Diagnosis #1 : Medication-induced ED

History finding(s)
. Started beta blocker 4 months ago
. No nighttime erection
. Difficulty having daytime erection

Physical examination finding(s)
. None

Diagnosis #2 : testosterone deficiency

History finding(s)
. Fatigue
. Erectile dysfunction

Physical examination finding(s)
. None

Diagnosis #3 : Anxiety

History finding(s)
. History of anxiety
. Increased stress over past 6 months

Physical examination finding(s)
. No focal findings on examination

Diagnostic studies
. Serum glucose and hemoglobin A1c
. Serum testosterone and TSH







µoªí¤å³¹®É¶¡2018/08/15 03:28pm¡@IP: ¤w³]©w«O±K[¥»¤å¦@ 7584 ¦ì¤¸²Õ]¡@ 
JuanFe 

 

¸ê®Æ: ¦¹·|­û¥Ø«e¤£¦b½u¤W Male ¨°Às ¥Õ¦Ï®y
«Â±æ: 0
¾y¤O: ¾y¤O: 78912
¸gÅç: ¸gÅç: 35968
¨Ó¦Û: ¦t©z¤¤¡@blank
µo¤å: 1118 ½g
ºëµØ: 0 ½g
¦b½u: 47¤Ñ19®É11¤À43¬í
µù¥U: 2013/06/17
Message ¬d¬Ý¡@·j´M¡@³q°T¿ý¡@½Æ»s¡@¤Þ¥Î¡@¦^ÂФ峹¦^ÂС@[²Ä 13 ¼Ó]
 12 Case 12 scenario (child fever)

Doorway information about patient

You will be speaking with the mother of a 1-year-old with fever

Basic differential diagnosis

. Respiratory tract infection
. Ear infection
. Exanthematous disease
. Meningitis
. Urinary tract infection
. Gastroenteritis

¡X¡X¡X¡X¡X

Case 12  sim.pt. instruction

This is a telephone encounter with the doctor , who is asking you questions on the phone input the child . If the doctor asks you about anything other than these , Just say ¡§no,¡¨ or provide an answer that a normal patient might give.

You are a 28-year-old woman who calls for evaluation of fever in your 1-year-old child

History of present illness

. Onset of fever 2 days ago
. Fever to 38.9¡¦C (102F)
. Associated symptoms include dry cough , picking at the right ear , and vomiting ingested food without diarrhea
. Had 1 episode of rhythmic jerking in both arms , with loss of urine ; afterward was silent and irritable for few minutes an then slept
. Dose met appear drowsy or lethargic but has not been feeling well
. Acetaminophen decreases the fever to 37.2¡¦C (99F) transiently ; tepid sponge bathes also provide some relief
. 3 year old sibling has no symptoms
. No recent travel

Review of systems

. No rash
. Passing normal yellow urine but cries while urinating

Past medical history

. No prior medical conditions , surgeries , or hospitalizations
. Full-term delivery without complications
. Breastfed until 2 months old , then changed to formula
. Able to stand , hold objects in the hand , and say ¡§Mama¡¨ and ¡§ Dada¡¨
. Childhood vaccinations are up to date
. At 9-month well-child visit , was at 75 percentile for height and 60 percentile for weight
. Chile lives at home with mother , father , and 3-year-old sibling
. Medications : Acetaminophen , liquid as needed
. Allergies : None
. Parents and siblings are all healthy

¡X¡X¡X¡X¡X

Case 12 sim. pt. checklist

Following the encounter , check which of the following items were performed by the examinee

History of present illness/ review of systems

. Asked about the onset and severity of fever
. Asked whether the fever is continuous or intermittent with spontaneous return to baseline
. Asked  whether the fever responds to any medications
. Asked about associated symptoms , including:
- Cough
- Ear discharge
- Nasal congestion / discharge
- Pain
- vomiting and diarrhea / bowel symptoms
- Urinary symptoms
- Rash
. Asked about association with seizures (and further details including description , onset , associated incontinence or urine / bowel)
. Asked about what happened after seizure : whether the child was irritable and if any body part was paralyzed
. Asked about any exposure to infected individuals and history of travel

Past medical /family/social history

. Asked about similar episodes i the past
. Asked about current medications and allergies
. Asked about past medical issues (ear infections , convulsions, urinary tract infections),surgeries , and hospitalizations
. Asked about motor and social development
. Asked about prenatal and perinatal history
. Asked about feeling habits
. Asked about family health (especially seizures)

Counseling

. Explained the possible diagnosis
. Explained further workup
. Advised to give fluid and antipyretic
. Discussed the need for a clinic visit for examination and testing

Communication skills and professional conduct

. introduced self and greeted you warmly
. Used your name to address you
. Paid attention to what you said and maintained good eye contact
. Asked opened questions
. Asked non-leading questions . Asked one question at a time
. Listened to what you said without interrupting me
. Used plain English rather than technical jargon
. Used appropriate transition sentences
. Expressed empathy and gave appropriate reassurances
. Asked whether you have any concerns/questions

Differential diagnosis

. Febrile seizures
. Meningitis
. Acute otitis media
. Urinary tract infection

Diagnostic study/studies

. CBC with differential count and erythrocyte sedimentation rate
. Urinalysis
. Lumber puncture

¡X¡X¡X¡X¡X

Case 12 clinical summary

Clinical Skills Evaluation
Case 12 Patient Note

The following represents a typical note for this patient encounter . the detail may vary depending on the information given by the simulated patient.

History : Describe the history you just obtained form this patient . Include only information (Pertinent positives and negatives)  relevant to this patient¡¦s problem(s).

. Telephone encounter with motor of 1-yo child with 2 days of fever to 38.9¡¦C (102F),cough , and pulling at the right ear
. 2 episodes of vomiting but no diarrhea.
. 1 episodes of vomiting but no diarrhea.
. Crying when passing yellow urine
. No lethargy
. No previous infections or hospitalizations
. No sick contacts; sibling has no symptoms
. Birth history remarkable, normal developmental milestones.
. Immunizations up to date.

ROS : No recent travel or rashes
PMHx : None
PSHx : None
Meds : Acetaminophen
Allergies : None
FHx : Parents and sibling are healthy

Physical examinations : Describe any positive and  negative findings relevant  to this patient¡¦s problem(s) . be careful to include only those parts of the examination performed in this encounter.

. Not done as this is a telephone encounter

Data interpretation : Based on what you have learned from the history and physical examination, list up to 3 diagnosis that might explain this patient¡¦s complaint(s) . List your diagnoses from most to least likely . For some cases , fewer than 3 diagnosis will be appropriate . the , enter the positive and negative findings form the history and the physical examination (if present) that support each diagnosis . Finally , list initial diagnostic (if any) you would order for each listed diagnosis (eg,restricted physical examination maneuvers, laboratory tests , imaging ECG , etc).

Diagnosis #1 : Acute otitis media

History finding(s)
. Fever
. Pulling at right ear

Physical examination finding(s)
. None

Diagnosis #2 : Urinary tract infection

History finding(s)
. Fever
. Pain with urination

Physical examination finding(s)
. None

Diagnosis #3 : Febrile seizure

History finding(s)
. Fever
. Rhythmic tremor with fever that subside
. Shaking movements while passing urine

Physical examination finding(s)
. None

Diagnostic studies
. Office visit for ear examination
. Urinalysis
. CBC with differential








µoªí¤å³¹®É¶¡2018/08/15 03:30pm¡@IP: ¤w³]©w«O±K[¥»¤å¦@ 6800 ¦ì¤¸²Õ]¡@ 
JuanFe 

 

¸ê®Æ: ¦¹·|­û¥Ø«e¤£¦b½u¤W Male ¨°Às ¥Õ¦Ï®y
«Â±æ: 0
¾y¤O: ¾y¤O: 78912
¸gÅç: ¸gÅç: 35968
¨Ó¦Û: ¦t©z¤¤¡@blank
µo¤å: 1118 ½g
ºëµØ: 0 ½g
¦b½u: 47¤Ñ19®É11¤À43¬í
µù¥U: 2013/06/17
Message ¬d¬Ý¡@·j´M¡@³q°T¿ý¡@½Æ»s¡@¤Þ¥Î¡@¦^ÂФ峹¦^ÂС@[²Ä 14 ¼Ó]
 13 Case 13 scenario (abdominal pain RUQ)

Doorway information about patient

The patient is a 45-year-old woman who comes to the office due to acute right upper quadrant abdominal pain.

Vital signs
. Temperature ; 38.3¡¦C (101F)
. Blood pressure : 130/80 mmHg
. Pulse ; 100/min
. respirations : 20/min

Basic differential diagnosis

Gastrointestinal . Acute cholecystitis
. Biliary colic
. Acute hepatitis
. Peptic ulcer (perforation)
. Acute pancreatitis (biliary pain)

Pulmonary
. Right lower lobe pneumonia

Cardiovascular
. Myocardial infarction
. Heart failure with hepatic congestion

Miscellaneous
. Herpes zoster (shingle)

¡X¡X¡X¡X¡X

Case 13 sim.pt. instruction

If the doctor asks you about anything other than these , just say ¡§ no ,¡¨ or provide an answer that a normal patient might give.

You are a 45-year-old woman with acute abdominal pain

History of present illness

. Sudden onset 2 hour ago , 30 minutes after eating
. Progressively worsening
. Right upper abdomen with radiation to back and right shoulder
. Severity 8-9/10
. ¡§Stabbing¡¨ sensation
. Worse with deep breathing , not relieved with antacids
. Associated symptoms : - Nausea and vomiting without blood or bile; feel warm , but you idid not check temperature
- No diarrhea
. Similar pain 3-4 times over the last 5 months ; usually after meals and sometimes better with antacids

Do not volunteer this information unless asked about diet or fatty foods : You eat a lot of fast food because you are busy at work and do not have time to cook.

Review of systems

. No jaundice , cough , shortness of breath , itching , or chest pain

Past medical history

. No prior medical problems
. C-section 20 years ago
. Medications : Over -the -counter antacids
. Allergies : None
. Immediate family members are healthy
. Occupation ; Accountant
. Married , live with husband and 1 child
. Tobacco : 1 pack of cigarettes a day for 25 years ; trying to cut down
. Alcohol ; 2-3 beers a day for 15 years
. Recreational drugs : No

Physical examination

Abdomen:
. Right upper quadrant discomfort with deep palpation ; slightly worse with deep breath
. Abdomen non-distended
. Normative bowel sounds throughout
. No hepatosplenomegaly
. No abdominal bruits

The remainder of the examination is normal.

¡X¡X¡X¡X¡X

Case 13 sim. pt. checklist

Following the encounter , check which of the following items were performed by the examinee

History of present illness/ review of systems

. Asked about the location and radiation of pain
. Asked about the intensity of pain
. Asked about the quality of pain
. Asked about the onset and duration of pain
. Asked about the precipitating factors
. Asked about the course of pain over time
. Asked about any aggravating or relieving factors
. Asked about nausea and vomiting
. Asked about fever and chills
. Asked about cough and breathing problems
. Asked about any chest pain
. Asked about jaundice
. Asked about history of black stools

Past medical /family/social history

. Asked about similar episodes in the past
. Asked about past medial issues (acid peptic disease , gallstone , heart problems)
. Asked about previous hospitalizations and surgeries (especially gallbladder removal or appendectomy)
. Asked about family history of healthy issue (especially gallstone)
. Asked about current medications
. Asked about occupation
. Asked boy tobacco and alcohol use
. Asked about diet

Examination

. Washed heads before examination
. Examined without gown , not through gown
. Auscultated abdomen(before palpation)
. Palpated abdomen (Superficial and deep)
. Checked for rebound tenderness
. Percussed for liver span
. Elicited murphy sign
. Auscultated heart and lungs

Counseling

. Explained physical findings and possible diagnosis
. Explained further workup
. Discussed change in lifestyle , including quitting smoking , cutting down alcohol , healthier diet

Communication skills and professional conduct

. Knocked before entering the room
. Introduced self and greeted you warmly
. Used your name to address you
. Paid attention to what you said and maintained good eye contact
. Asked opened questions
. Asked non-leading questions . Asked one question at a time
. Listened to what you said without interrupting me
. Used plain English rather than technical jargon
. Used appropriate transition sentences
. Used appropriate draping techniques
. Summarized the history and explained physical findings
. Expressed empathy and gave appropriate reassurances
. Asked whether you have any concerns/questions

Differential diagnosis

. Acute cholecystitis
. Biliary colic
. Perforation of peptic ulcer
. Pancreatitis
. Acute hepatitis

Diagnostic study/studies

. CBC with differential count
. EKG
. Chest x-ray
. Ultrasound abdomen
. Serum amylase and lipase
. LFTs (albumin , AST,ALT, alkaline phosphatase , total and direct bilirubin)

¡X¡X¡X¡X¡X

Case 13 clinical summary

Clinical Skills Evaluation
Case  Patient Note

The following represents a typical note for this patient encounter . the detail may vary depending on the information given by the simulated patient.

History : Describe the history you just obtained form this patient . Include only information (Pertinent positives and negatives)  relevant to this patient¡¦s problem(s).

. 45-yo female with 2 hours of worsening RUQ abdominal pain radiating to the right scapula.
. 5 months of similar episodes (3-4/month) that resolved with antacids
. Stabbing pain starting 30 minutes after food with nausea and nonbiilious and non bloody vomitus.
. Pain worse with deep breathing and not improved with antacids.

ROS : No jaundice , cough , shortness of breath , itching , chest pain , or diarrhea
PMHx : None
PSHx : Cesarian delivery 20 years ago
Meds : OTC antacids PRN
Allergies : None
FHx : Parents and siblings are healthy
SHx : 1 PPD smoker for 25 years , 2 or 3 beers /day for 15 years

Physical examinations : Describe any positive and  negative findings relevant  to this patient¡¦s problem(s) . be careful to include only those parts of the examination performed in this encounter.

. Vital signs :Temperature, 38.3¡¦C (101.0F) ; blood pressure , 130/80 mmHg ; pulse , 100/min ; and respirations , 20/min
. HEENT : PERRLA, EOMI, no jaundice
. Abdomen : RUQ discomfort with deep palpation ; non-distended , normative bowel sounds throughout ; no hepatosplenomegaly or bruits

Data interpretation : Based on what you have learned from the history and physical examination, list up to 3 diagnosis that might explain this patient¡¦s complaint(s) . List your diagnoses from most to least likely . For some cases , fewer than 3 diagnosis will be appropriate . the , enter the positive and negative findings form the history and the physical examination (if present) that support each diagnosis . Finally , list initial diagnostic (if any) you would order for each listed diagnosis (eg,restricted physical examination maneuvers, laboratory tests , imaging ECG , etc).

Diagnosis #1 : Acute cholecystitis

History finding(s)
. RUQ abdomen pain
. 5 month of similar episodes
. Pain radiating to right shoulder
. Pain worsened with deep breathing

Physical examination finding(s)
. Fever
. RUQ tenderness
. Positive Murphy sign

Diagnosis #2 : Acute pancreatitis

History finding(s)
. RUQ pain
. Nausea and vomiting
. Alcohol use

Physical examination finding(s)
. RUQ tenderness
. Fever

Diagnosis #3 : Peptic ulcer

History finding(s)
. Nausea and vomiting
. RUQ pain
. Alcohol /tobacco use

Physical examination finding(s)
. Fever
. RUQ tenderness

Diagnostic studies
. Ultrasound of RUQ of abdomen
. Serum amylase and lipase
. Liver function tests
. CBC with differential







µoªí¤å³¹®É¶¡2018/08/15 03:31pm¡@IP: ¤w³]©w«O±K[¥»¤å¦@ 8162 ¦ì¤¸²Õ]¡@ 
JuanFe 

 

¸ê®Æ: ¦¹·|­û¥Ø«e¤£¦b½u¤W Male ¨°Às ¥Õ¦Ï®y
«Â±æ: 0
¾y¤O: ¾y¤O: 78912
¸gÅç: ¸gÅç: 35968
¨Ó¦Û: ¦t©z¤¤¡@blank
µo¤å: 1118 ½g
ºëµØ: 0 ½g
¦b½u: 47¤Ñ19®É11¤À43¬í
µù¥U: 2013/06/17
Message ¬d¬Ý¡@·j´M¡@³q°T¿ý¡@½Æ»s¡@¤Þ¥Î¡@¦^ÂФ峹¦^ÂС@[²Ä 15 ¼Ó]
 14 Case 14 scenario (prenatal visit)

Doorway information about patient

The patient is a 24-year-old woman who comes to the office for an initial prenatal visit

Vital signs
. Temperature : 37.1¡¦C(98.8F)
. Blood pressure : 120/75 mmHg
. Pulse : 78/min
. Respirations : 20/min

¡X¡X¡X¡X¡X

Case 14 sim.pt. instruction

If the doctor asks you about anything other than these , just say ¡§ no ,¡¨ or provide an answer that a normal patient might give.

You are a 25-year-old woman here for your first prenatal visit

. This is your first pregnancy (no prior miscarriage or abortion)
. Last menstrual period 12 weeks ago
. Positive home pregnancy test 6 weeks ago
. Have not felt any fetal movements yet
. Menarche at age 13 ; periods usually regular every 28-30 days with bleeding 4-5 days each month
. No morning sickness , vomiting , abdominal pain , vaginal bleeding , fever , rash , breathing problems , sleep disturbances , or swelling in the feet

Past medical history

. No prior medical problems
. No surgeries or hospitalizations
. Medications : None
. Medication allergies : None
. Up to date on all standard adult immunizations
. Immediate family members are all healthy
. Occupation : Homemaker
. Married , lived with husband
. Tobacco 1 pack a day for 5 years
. Alcohol : 1-3 beers a week for 3 years
. Recreational drugs : None

Physical examination

HEENT

. PERRLA, EOMI
. Oropharynx clear

Lungs : . Clear to auscultation bilaterally

Heart : . Regular rate and rhythm without murmurs, gallops , or rubs

Abdomen :

. Non-tender , Non-distended
. Normative bowel sounds throughout
. No hepatopslenomegaly
. No bruits

Extremities

. No cyanosis or edema

¡X¡X¡X¡X¡X

Case 14 sim. pt. checklist

Following the encounter , check which of the following items were performed by the examinee

History of present illness/ review of systems

. Asked about LMP
. Asked about how you first suspected and confirmed pregnancy
. Asked about obstetrical / reproductive history (pregnancy and abortions)
. Asked about gynecologic history (menarche, regular periods , contraception)
. Asked about any pregnancy -related problems (vomiting , fever , abdominal pain, vaginal bleeding)
. Asked about diet , appetite , and weight gain
. Asked about genitourinary symptoms (eg , discharge, lesions, dysuria)
. Asked about sleep

Past medical /family/social history

. Asked about past medical issues (especially heart conditions , autoimmune disorders , hypertension , diabetes , sexually transmitted infections ,ad n renal disease)
. Asked about previous blood transfusions
. Asked bit current medications
. Asked about medication allergies
. Asked about exposure to cats
. Asked about rubella immunization in the past
. Asked about family health (congenital or birth problems in the family)
. Asked about tobacco , alcohol , and drug use
. Asked about occupation

Examination

. Washed heads before examination
. Examined without gown , not through gown
. Examined eyes (for pallor)
. Examined oral cavity (for general hygiene)
. Examined legs (for edema and varicose viens)
. Auscultated heart and lungs
. Examined and auscultated abdomen (if <28 weeks of fundal grip ; if > 28 weeks do Leopold maneuvers)

Counseling

. Explained physical findings
. Discussed appropriate prenatal tests
. Advised you to stop usage of tobacco and alcohol
. Advised safe sexual practices
. Explained the need for prenatal vitamins , iron supplementation , and nutritious diet
. Explained the importance of regular antenatal visits

Communication skills and professional conduct

. Knocked before entering the room
. introduced self and greeted you warmly
. Used your name to address you
. Paid attention to what you said and maintained good eye contact
. Asked opened questions
. Asked non-leading questions . Asked one question at a time
. Listened to what you said without interrupting me
. Used plain English rather than technical jargon
. Used appropriate transition sentences
. Used appropriate draping techniques
. Summarized the history and explained physical findings
. Expressed empathy and gave appropriate reassurances
. Asked whether you have any concerns/questions

Diagnostic study/studies

. RH(D) blood type , antibody screen
. CBC (hemoglobin /hematocrit , MCV)
. HIV , VDRL/RPR, HBsAg
. Rubella and varicella titers
. Pelvic examination (with Pap test, if indicated)
. Chlamydia PCR
. Urinalysis and culture

¡X¡X¡X¡X¡X

Case 14 clinical summary

Clinical Skills Evaluation
Case 14 Patient Note

The following represents a typical note for this patient encounter . the detail may vary depending on the information given by the simulated patient.

History : Describe the history you just obtained form this patient . Include only information (Pertinent positives and negatives)  relevant to this patient¡¦s problem(s).

. 24-yo woman , G1P0A0 and LMP 12 weeks ago , who presents for her first prenatal visit.
. No fetal movements yet.

ROS : No mooning sickness , vomiting , abdominal pain , vaginal bleeding , fever , rash , breathing problems , sleep disturbances , or swelling in the feet
PMHx : None
PSHx : None
Meds : None
Allergies : None
FHx : parents and siblings are healthy
SHx : 1 PPD smoker for 5 years , 1-3 beers/ week fro 3 years

Physical examinations : Describe any positive and  negative findings relevant  to this patient¡¦s problem(s) . be careful to include only those parts of the examination performed in this encounter.

.Vital signs ; temperature , 37.1 ¡¥C (98.8F) ; blood pressure , 120/75 mmHg ; pulse , 78 /min; and respirations , 20/min
. HEENT : PERRLA < EOMI , no jaundice , oropharynx clear
. Lungs : Clear to auscultation bilaterally
. Heart : RRR without murmurs, gallops , or rubs
. Abdomen : Non-tender , non-distended , normative bowel sounds throughout ; no hepatosplenomegaly ; no bruits
. Extremities : No cyanosis or edema

Data interpretation : Based on what you have learned from the history and physical examination, list up to 3 diagnosis that might explain this patient¡¦s complaint(s) . List your diagnoses from most to least likely . For some cases , fewer than 3 diagnosis will be appropriate . the , enter the positive and negative findings form the history and the physical examination (if present) that support each diagnosis . Finally , list initial diagnostic (if any) you would order for each listed diagnosis (eg,restricted physical examination maneuvers, laboratory tests , imaging ECG , etc).

Diagnosis #1 : Normal pregnancy

History finding(s)
. LMP 12 weeks ago
. Home pregnancy test positive 6 weeks ago

Physical examination finding(s)
. non-distended abdomen
. No edema

Diagnostic studies
. Rh(D)type , antibody screen
. CBC
. HIV, VDRL/RPR , HBsAg
. Rubella and varicella titers
. Pelvic examination
. Chlamydia PCR
. Urinalysis and culture







µoªí¤å³¹®É¶¡2018/08/15 03:31pm¡@IP: ¤w³]©w«O±K[¥»¤å¦@ 7201 ¦ì¤¸²Õ]¡@ 

 ¦@ 3 ­¶ 9 7 [ 1 2 3 ] 8 :

§Ö³t¦^ÂÐ¥DÃD: Sample cases
±z¥Ø«eªº¨­¥÷¬O¡G ³X«È ¡A­n¨Ï¥Î¨ä¥L·|­û¨­¥÷¡A½Ð¿é¤J·|­û¦WºÙ©M±K½X¡C¥¼µù¥U³X«È½Ð¿é¤Jºô¦W¡A±K½X¯dªÅ¥Õ¡C
¿é¤J·|­û¦WºÙ©M±K½X: ·|­û¦WºÙ: ¨S¦³µù¥U¡H¡@±K½X: §Ñ°O±K½X¡H
¤W¶Çªþ¥ó©Î¹Ï¤ù (³Ì¤j®e¶q 9000KB)
¥Ø«eªþ¥ó:(¦p¤£»Ý­n¬Y­Óªþ¥ó¡A¥u»Ý§R°£¤º®e¤¤ªº¬ÛÃö [UploadFile ...] ¼ÐÅÒ§Y¥i) [§R°£]
¿ï¶µ

¨Ï¥Î LeoBBS ¼ÐÅÒ¡H
Åã¥Ü±zªºÃ±¦WÀÉ¡H
¨Ï¥Îªí±¡²Å¸¹Âà´«¡H
¨Ï¥Î¦r«¬¼Ë¦¡Âà´«¡H

¡@¡@¡@¡@§Ö³t¤Þ¥Î²Ä ¼Ó¼hªº¦^ÂÐ
 ³»ºÝ¡@¥[¨ì"§Úªº³Ì·R" ¥DÃDºÞ²z¡G Á`©T³» ¨ú®øÁ`©T³» °Ï©T³» ¨ú®ø°Ï©T³» ©T³» ¨ú®ø©T³»
¥[­« ¨ú®ø¥[­« ºëµØ ¨ú®øºëµØ ´£¤É Âê©w ¸ÑÂê §R°£ §R°£¦^ÂÐ ²¾°Ê


© ¤¤¤åª©Åv©Ò¦³¡G ¶ð¤º´Óª«¶é¡@ÁcÅ骩Åv©Ò¦³¡G ¼¯¥§ºô
µ{¦¡ª©Åv©Ò¦³¡G ¹p¶Æ¬ì§Þ¡@ µ{¦¡Â½Ä¶¡Gauron¡@ª©¥»¡G LeoBBS X Build041101
 

¥»½×¾Â¨¥½×¯ÂÄݵo¨¥ªÌ­Ó¤H·N¨£¡A»P ¶ð¤º´Óª«¶é ¥ß³õµLÃö